Chest Pain

Ace your homework & exams now with Quizwiz!

Essential

Most hypertension cases are classified as ____________________

65 years old

What is the age that you check for an Aortic aneurism who smoked at any point of their life

E. Aspirin and Corticosteroids

A 2-year-old boy comes to the emergency department because of high fever and rash for five days. His pediatrician saw him two days ago and felt his fever and rash were likely due to a non-specific viral infection. Since that appointment, he had been refusing to eat solids and had only been taking sips from his bottle and cups. His temperature is 39.6°C (103.2°F). Examination shows conjunctival injection bilaterally. His tongue is bright red and edematous. His tonsils are small but erythematous and without exudate. There is a 1.5-cm (0.59-in) lymph node along the left anterior cervical chain and a 2-cm (0.79-in) supraclavicular lymph node on the right. He has a diffuse, erythematous maculopapular type rash on his chest, abdomen, and back extending into the groin region. His hands and feet are red and painful to touch. Which Medication options will help? A. Indomethacin and Aspirin B. Corticosteroids C. Aspirin D. Indomethacin E. Aspirin and Corticosteroids F. Prostaglandin E1 and E2 G. Prostaglandin E1 and E2 and Corticosteroids

A. Kawasaki disease Major takeaway Kawasaki disease (KD) is a relatively common, self-limited vasculitis that affects young children. It is characterized by high fever, conjunctivitis, rash, cervical adenopathy, and oral cavity changes. Main explanation Kawasaki disease (KD) is a relatively common, self-limited vasculitis that affects young children. Fever >39°C (102.2°F) of over 5 days duration is required for the diagnosis of KD. Parents will also often provide a history of intense irritability with the fever. Additionally, this child has all of the 5 characteristics that are also required for the diagnosis. They are described below: Changes in extremities - manifest as edema, erythema and desquamation.The hands and feet are often painful and may result in the children refusing to bear weightConjunctivitis - frequently bilateralRash - there is no classic rash with KD. It may be erythematous, maculopapular, or occasionally reticular and sometimes limited to the groin and extremities Cervical adenopathy - at least one node greater than 1.5 cm in diameterChanges in oral cavity - dry, cracked lips, pharyngeal edema,"strawberry tongue"

A 2-year-old boy comes to the emergency department because of high fever and rash for five days. His pediatrician saw him two days ago and felt his fever and rash were likely due to a non-specific viral infection. Since that appointment, he had been refusing to eat solids and had only been taking sips from his bottle and cups. His temperature is 39.6°C (103.2°F). Examination shows conjunctival injection bilaterally. His tongue is bright red and edematous. His tonsils are small but erythematous and without exudate. There is a 1.5-cm (0.59-in) lymph node along the left anterior cervical chain and a 2-cm (0.79-in) supraclavicular lymph node on the right. He has a diffuse, erythematous maculopapular type rash on his chest, abdomen, and back extending into the groin region. His hands and feet are red and painful to touch. Which of the following is the most likely diagnosis? A. Kawasaki disease B. Measles C. Rubella D. Scarlet fever E. Toxic epidermal necrosis

E. P-ANCA Major takeaway Eosinophilic granulomatosis with polyangiitis (Churg-Strauss syndrome) is an ANCA-associated (p-ANCA) small vessel vasculitis that is characterized by eosinophilia, asthma, and vasculitis. Skin findings include blisters and bleeding areas with a central black dimple representing tissue necrosis. Main explanation Eosinophilic granulomatosis with polyangiitis (EGPA; also known as Churg-Strauss syndrome [CSS] or allergic granulomatosis) is a rare autoimmune condition that causes vasculitis in persons with a history of airway allergic hypersensitivity (atopy). CSS is characterized by three stages: the early (prodromal) stage is marked by airway inflammation; almost all patients experience asthma and/or allergic rhinitis. The second stage is characterized by hypereosinophilia, which causes tissue damage, most commonly to the lungs and the digestive tract. The third stage consists of vasculitis, which can eventually lead to cell death and can be life-threatening. Laboratory analysis shows eosinophilia and raised immunoglobulin E levels. 50-60 % of patients are ANCA positive, predominantly p-ANCA. Rheumatoid factor is also positive in 70 % of patients. The American College of Rheumatology 1990 criteria for diagnosis of Churg-Strauss syndrome lists these criteria: asthma, eosinophils greater than 10% of a differential white blood cell count, mononeuropathy or polyneuropathy, unfixed pulmonary infiltrates, paranasal sinus abnormalities, and histological evidence of extravascular eosinophils. Treatment for CSS includes glucocorticoids (e.g. prednisolone) and other immunosuppressive drugs (e.g. azathioprine and cyclophosphamide). In many cases, the disease can be put into a type of chemical remission through drug therapy, but the disease is chronic and lifelong.

A 33-year-old man comes to the clinic because of new skin lesions for the past 9 months. The patient says that the skin lesions have been growing in number and that they tend to blister and then form a black center. Medical history is contributory for asthma for which he requires an albuterol inhaler as needed. Physical examination shows bleeding areas on the arms and legs with a central black dimple. Laboratory test shows 11% of eosinophils in the blood. Which of the following serologies is most likely to be positive or elevated in this patient's condition? A. Anti-Ro B. Anti-Sm C. C-ANCA D. HBsAg E. P-ANCA

E. Serum lipase > 3 times the upper limit of normal Major takeaway Acute pancreatitis is characterized by epigastric abdominal pain radiating to the back, often with nausea and vomiting. The diagnosis is made by meeting 2 of the 3 clinical, laboratory, and imaging criteria. The preferred diagnostic laboratory test for acute pancreatitis is a serum lipase greater than 3 times the upper limit of normal. Main explanation This patient is presenting with constant abdominal pain localized in the epigastric region that radiates to the back, highly suggestive of acute pancreatitis. Acute pancreatitis occurs when pancreatic enzymes are inappropriately activated, leading to tissue damage and an inflammatory response. This patient has a family history of recurrent pancreatitis in his father and paternal uncle, suggesting the diagnosis of familial hypertriglyceridemia, an autosomal dominant condition. The high concentration of serum triglycerides can cause acute pancreatitis because when pancreatic cells encounter triglycerides, they release lipase to break the triglycerides down into free fatty acids; excess free fatty acids can subsequently injure the pancreas. The most common causes of pancreatitis are gallstones and alcohol toxicity. The other causes of pancreatitis can be easily remembered by the mnemonic "I GET SMASHED":

A 44-year-old man comes to the emergency department with abdominal pain and nausea for the past 24 hours. The pain started a few hours after dinner, and he describes it as a constant pain in the upper part of his abdomen that radiates to his back and flanks. The patient also reports nausea, intermittent vomiting, and abdominal distention. Family history is significant for recurrent episodes of pancreatitis in his father and paternal uncle. He reports drinking an average of 2 alcoholic drinks per week. His temperature is 37.7ºC (99.9ºF), pulse is 92/min, respirations are 20/min, peripheral oxygen saturation is 96%, and blood pressure is 149/94 mmHg. He appears distressed and anxious. Abdominal examination shows tenderness and rigidity over the epigastric and periumbilical regions. Bowel sounds are diminished. No bruising of the umbilicus or flanks are seen. Which of the following laboratory results is the most specific in diagnosing this patient's condition? A. Serum amylase > 3 times the upper limit of normal B. Serum bilirubin > 4.0 mg/dL C. Blood urea nitrogen (BUN) level > 22 mg/dL D. Elevated blood alcohol leve E. Serum lipase > 3 times the upper limit of normal

B. Elevated creatine kinase-MB concentration Major takeaway Myocardial infarction causes elevated concentrations of troponin and creatine kinase-MB (CK-MB). Plasma CK-MB concentrations typically rise about 4-6 hours after the onset of chest pain. Main explanation The patient is currently suffering from acute coronary syndrome (ACS), specifically a non-ST-segment elevation myocardial infarction (NSTEMI). It's most commonly caused by a reduction in oxygen supply and/or by an increase in myocardial oxygen demand superimposed on a lesion that causes coronary arterial obstruction. This is usually an atherothrombotic coronary plaque. In the setting of myocardial infarction, plasma creatine kinase-MB (CK-MB) concentrations typically rise about 4-6 hours after the onset of chest pain. They peak within 12-24 hours and return to baseline concentrations within 24-48 hours. In ST-segment elevation myocardial infarction (STEMI) and NSTEMI this cardiac biomarker will rise in blood along with troponin; however, in patients with unstable angina (UA), CK-MB will not rise and troponin will not be present.

A 64-year-old man comes to the emergency department because of chest discomfort for the past 5 hours. When the patient is asked where the pain is located, he places a clenched fist to indicate a squeezing over his substernal region. The pain radiates to the neck, left shoulder, and left arm. His temperature is 36.8°C (98°F), pulse is 55/min, respirations are 17/min, and blood pressure is 117/78 mm Hg. Examination shows a diaphoretic male. ECG shows inverted U-waves in leads V5 and V6. The cardiac biomarker test shows an elevated concentration of troponin. Which of the following is most likely consistent with this patient's diagnosis? A. Diffuse ST-segment elevation B. Elevated creatine kinase-MB concentration C. Q-wave on ECG D. ST-segment elevation on ECG E. Troponin I is normal while troponin T is highly elevated

E. Gastric ulcer Major takeaway Gastric ulcers are characterized by epigastric pain shortly after eating and are most commonly due to H. Pylori infection or NSAID use. Main explanation Peptic ulcer disease (PUD) is used to describe both gastric and duodenal ulcers. Although differentiation between the two is difficult on history alone there are some clues that may assist. Gastric ulcers most often occur in the antrum of the stomach and lead to pain within 15-30 minutes of eating. There is generally no relief from antacids. The pain from duodenal ulcers typically occurs 2-3 hours after eating and is relieved by food and antacids.Helicobacter pylori infection and NSAID usage cause the majority of peptic ulcers. It is important to test for H. Pylori in all patients with suspected PUD as it is easily treated with oral therapy. If patients with PUD require ongoing NSAID therapy gastroprotection using a proton pump inhibitor is recommended.

A 68-year-old man comes to the clinic because of recurrent pain "right above the belly button" and nausea. His pain occurs less than an hour after he's eaten. The only medications he reports taking are preventive low-dose aspirin and vitamin supplements. He is otherwise healthy. He has tried over-the-counter antacids for the pain with no benefit. Which of the following types of ulcer is most likely to be present in this patient? A. Colonic ulcer B. Curling ulcer C. Duodenal ulcer D. Esophageal ulcer E. Gastric ulcer

(B) Fibrinous pericarditis Answer B (correct answer): This is a typical presentation of peri-infarction pericarditis (fibrinous pericarditis). Myocardial inflammation after myocardial infarction (MI) sometimes involves the pericardium leading to pericarditis. This occurs within the first few days following MI. The pain of pericarditis is usually present at rest. It worsens with lying flat and improves with sitting upright. The pain is often worse with inspiration (pleuritic). Cardiac exam may reveal a friction rub. Answer A (incorrect answer): Cardiac tamponade occurs when a large pericardial effusion limits ventricular filling. This patient has a small pericardial effusion - a common finding in pericarditis. In addition, she has no change in blood pressure with inspiration indicating absence of a pulsus paradoxus, the classic hallmark of tamponade. Answer C (incorrect answer): Systolic heart failure causes dyspnea, fatigue, and edema. Lungs will have rales. An S3 is often present. Jugular venous pressure is usually elevated. Answer D (incorrect answer): Pulmonary embolism may cause pleuritic chest pain as seen in this case. In addition, hospitalized patients on bed rest are at risk due to lack of mobility which causes stasis of blood. However, pulmonary embolism does not cause a friction rub. Answer E (correct answer): Dressler's syndrome is an autoimmune form of pericarditis. It can occur following MI but onset is 4 to 6 weeks later.

A 70-year-old woman develops chest pain. Three days ago she was treated for an anterior myocardial infarction with implantation of a drug-eluting stent to the left anterior descending artery. She felt well after stent implantation but developed new chest pain this morning. The pain is sharp and centrally located. It worsens with a deep breath. Blood pressure is 120/80 mmHg without change during respiration. Pulse is 80/min. Lungs are clear. On cardiac exam, S1 and S2 are normal. There is no S3 or S4. A friction rub is present. Jugular venous pressure is 7 mmHg (normal <8). ECG shows sinus rhythm with anterior Q waves. On echocardiogram, there is depressed left ventricular function, normal right ventricular function and a small pericardial effusion. Which of the following is the most likely diagnosis? (A) Cardiac tamponade (B) Fibrinous pericarditis (C) Acute systolic heart failure (D) Pulmonary embolism (E) Dressler's syndrome

Stops the conversion of Angiotensin 1 to 2

Why does the medication work

7 risk factors ¨Male sex ¨Age >55yrs. in men & >65 yrs. in women ¨Tobacco Use ¨Diabetes ¨Hypertension ¨Family History of premature CAD ¨Hyperlipidemia

¨Upon further questioning Mr. G who is 55 admits that he has been having symptoms for the past 5 months. He notices a squeezing chest pain on the left side of his chest while climbing stairs, painting his garage, and with occasional exercise. The pain goes away within 5 minutes of rest. His current medications include metformin and lisinopril. He has a 20-pack year smoking history. Diet consists of fatty foods. He has a family history for coronary artery disease. ¨How many risk factors for CAD does this patient have?

E Naproxen This young man is presenting with a pleuritic and reproducible chest pain. His pretest probability for acute coronary syndrome is quite low. Given his vigorous physical activity, physical exam findings, and normal chest x-ray, a musculoskeletal cause of chest pain is most likely. The patient's symptoms are suggestive of costochondritis, a musculoskeletal etiology of chest pain that responds well to NSAIDs. Alternative treatment modalities include hot/cold compresses and limiting painful activities. This condition is a common cause of chest pain in young patients with a history of vigorous physical exercise, as seen in this case.

A 26-year-old man comes to the physician because of a 1-week history of left-sided chest pain. The pain is worse when he takes deep breaths. Over the past 6 weeks, he had been training daily for an upcoming hockey tournament. He does not smoke cigarettes or drink alcohol but has used cocaine once. His temperature is 37.1°C (98.7°F), pulse is 75/min, and blood pressure is 128/85 mm Hg. Physical examination shows tenderness to palpation of the left chest. An x-ray of the chest is shown. Which of the following is the most appropriate initial pharmacotherapy? A Nitroglycerin B Alprazolam C Alteplase D Heparin E Naproxen

Major takeaway IgA vasculitis (Henoch-Schönlein purpura) is characterized by the classic clinical triad of palpable purpura petechiae, migratory arthritis, and abdominal pain. It usually occurs after a viral infection or streptococcal infection. Main explanation This child, who is presenting with palpable purpura/ petechiae , migratory arthritis, and abdominal pain that all occurred following an upper respiratory/throat infection, most likely has IgA vasculitis, formerly known as Henoch-Schönlein purpura (HSP). Other possible manifestations may include hematuria, proteinuria, intussusception, gastrointestinal hemorrhage, scrotal swelling and pain, as well as CNS and eye involvement. IgA-antibody immune complexes caused by antigenic exposure from an infection (such as a Streptococcal infection) or medication, deposit in the small vessels (usually capillaries) of the skin, joints, kidneys, and GI tract. This deposition results in an influx of inflammatory mediators such as prostaglandins. If the immune complexes are deposited in the intestinal wall, they may cause GI hemorrhage; intussusception is a major complication of HSP. Renal involvement of IgA-mediated immune complexes may result in crescentic glomerulonephritis. Immune complex deposits in the skin cause palpable purpura and petechiae. The management of HSP is mainly symptomatic and supportive, with therapy directed toward adequate oral hydration, bed rest, and symptomatic relief of joint and abdominal pain. Patients with joint pain can be treated with NSAIDs; in severe cases, oral steroids may be indicated.

A 6-year-old boy is brought to the clinician's office by his parent because of a 1-day history of rash, nausea, and stomach pain. The parent reports the patient had a throat infection several weeks ago which resolved spontaneously. A few days later, he developed a hip pain that subsided and was followed by right knee and ankle pain. At the same time, he developed a rash on his legs, buttocks, and forearms. Immunizations are up to date. Vitals are within normal limits. Physical examination reveals a rash, as shown in the image below. Musculoskeletal examination reveals a swollen right knee without effusion, erythema, or warmth. Scrotal swelling and sensitivity are noted on the left side. He has Abdominal pain. Urinalysis shows microscopic hematuria with proteinuria. Serum electrolytes, including creatinine, are within normal limits. Which of the following is the most likely diagnosis? A. Rheumatic fever B. IgA vasculitis C. Childhood-onset systemic lupus erythematosus D. Mumps infection E. Juvenile idiopathic arthritis

A. Abdominal aortic aneurysm Major takeaway Abdominal aortic aneurysms (AAA) classically present with a triad of hypotension, abdominal, low back, or flank pain, and a palpable pulsatile mass, although the findings of all three is a rare clinical occurrence. Primary risk factors include age >65 years, smoking, male gender, and a family history of AAA. Treatment should be offered to patients with AAA >5 cm, as risk of rupture increases exponentially beyond this value. Main explanation Abdominal aortic aneurysms (AAA) should be considered in any older individual presenting with sudden onset abdominal, low back, or flank pain and a history of syncope with hypotension. Risk factors for AAA include significant age >65 years, smoking, male gender, family history of AAA, and other risk factors for coronary artery disease or peripheral vascular disease including hypertension and hypercholesterolemia. Abdominal aortic aneurysms are true aneurysms, meaning all three layers of the blood vessel - the intima, media, and adventitia - are dilated. The pathophysiology of abdominal aortic aneurysms is a complex, multifactorial process that involves alterations in the structure of the vascular wall due to inflammatory changes, abnormal collagen remodeling and cross-linking, as well as loss of elastin and smooth muscle cells. This ultimately results in progressive thinning and weakening of the aortic wall and enlargement of the aortic diameter to a dilation 50% greater than the normal diameter of the aorta. AAAs commonly occur at the infrarenal level, or below the level of the renal arteries. People with near rupture or rapid expansion of a AAA commonly present with sudden onset back, flank, or abdominal pain and associated symptoms such as nausea and vomiting, mimicking a myriad of other conditions. Other associated symptoms may include diminished peripheral pulses due to expansion of the aneurysm compressing the internal iliac arteries. Although a pulsatile abdominal mass is a classic presentation, it may not always be detectable, especially if the patient is obese, such as in this vignette. If left untreated, AAAs can rupture, leading to severe hypotension, hemodynamic instability, and death. In a case where rupture has not occurred, repair should be offered when the aneurysm is >5 cm.

A 65-year-old man presents to the emergency department after a syncopal event at home. He has had two hours of left-sided flank pain radiating to the groin, which is associated with nausea and vomiting. Prior to this episode, he reports vague back pain and fatigue over the past several months. His past medical history is significant for an 80-pack-year smoking history, hypercholesterolemia, and hypertension. He has had no previous abdominal surgeries. His BMI is 30 kg/m2. His temperature is 37.0 °C (98.6 °F), pulse is 100/min, respirations are 20/min, blood pressure is 90/60 mmHg, and O2 saturation is 95% on room air. Abdominal examination shows distension and diffuse tenderness on palpation, and cardiovascular examination shows a 1+ dorsalis pedis pulse in the left foot and 2+ pulse in the right foot. Laboratory findings show a WBC of 12,000/mm3. Which of the following is the most likely diagnosis? A. Abdominal aortic aneurysm B. Cholangitis C. Bowel obstruction D. Acute pancreatitis E. Renal colic

LCX

Which Artery goes with the following lead: I, aVL

RCA

Which Artery goes with the following lead: II, III, aVF

LAD

Which Artery goes with the following lead: V1-V2

Distal LAD

Which Artery goes with the following lead: V3-V4

LAD or LCX

Which Artery goes with the following lead: V5-V6

PDA Posterior ST depression in V1-V3 with tall R waves

Which Artery goes with the following lead: V7-V9 Where are these leads located

D. Takayasu arteritis Major takeaway Takayasu arteritis affects the aortic arch and its branches. Asian women between the ages of 15-30 years are more commonly affected. Initial symptoms are nonspecific and they include malaise, fever, and weight loss. The obstruction of flow causes non-palpable pulses. The most accurate test is angiography. Treatment is with glucocorticoids. Main explanation Takayasu arteritis more commonly affects Asian women between the ages of 15-30 years. In some patients you may not be able to feel the pulse or measure the blood pressure on the affected side, hence, the name "pulseless disease". Initial symptoms are usually very nonspecific and they include malaise, fever, and weight loss. As the course of the disease progresses, vascular symptoms appear and dominate the clinical picture. Takayasu arteritis affects the aortic arch and its branches such as the cerebral, femoral, renal, mesenteric, and coronary vessels. The involvement of the carotid arteries may cause neurologic signs or stroke. Patients may also present with limb claudication and lightheadedness. When the disease is very severe, the obstruction of flow makes the pulses non-palpable. Narrowing of the coronary artery may lead to a myocardial infarction and involvement of the renal arteries may lead to uncontrolled systemic hypertension. The gold standard and most accurate test is angiography. Treatment is initially started with glucocorticoids. Cyclophosphamide, methotrexate, and TNF-alpha inhibitors are used to help remove patients from steroids. Patients that have very advanced stages of the disease may be treated surgically or treated with angioplasty.

A 25-year-old woman comes to the emergency department because of neck pain and transiently dimmed vision. A year ago, she developed a fever, fatigue, and bilateral neck pain. More recently, she has experienced dimming of the visual field in both eyes. She does not have episodes of imbalance, diplopia, or vertigo. She does not have any dyspnea or angina. Her temperature is 39.1°C (102.5°F) and pulse is 115/min. Her right arm blood pressure is 96/58 mm Hg but the left arm blood pressure could not be measured. Cardiac examination shows no abnormalities. Both of her carotid pulses and the right brachial pulse are weak. The left brachial pulse, right popliteal, and right dorsalis pedis pulses are not palpable. A bruit is audible over both carotid arteries and in the supraclavicular area on auscultation. Which of the following is the most likely cause of the patient's symptoms? A. Aortic dissection B. Kawasaki Disease C. Microscopic polyangiitis D. Takayasu arteritis E. Thromboangiitis obliterans

B. Aortic isthmus Major takeaway Traumatic rupture of the aorta is a life threatening injury that most commonly occurs during sudden deceleration related to motor vehicle accidents . The most common location of injury is the descending aorta at the isthmus. Main explanation Traumatic rupture of the aorta is a potentially life-threatening injury, with most cases occurring due to sudden deceleration related to automobile accidents . Approximately 80-90% of aortic tears occur in the descending aorta at the isthmus, which is just distal to the left subclavian artery and strongly tethered by the ligamentum arteriosum. Many patients will have retrosternal or scapular pain, and a widened mediastinum is visible on chest x ray. During deceleration, the more mobile aortic arch swings forward, producing a shearing force or "whiplash effect" on the aorta at the isthmus, which is fixed due to the attachment of the ligamentum arteriosum. This shearing force at the isthmus results in a tear. Less common sites of aortic trauma include the ascending aorta, the distal descending aorta at the level of the diaphragm, and the mid-thoracic descending aorta, because these areas of the aorta are more anatomically fixed and thus less subject to movement.

A 28-year-old man is brought to the emergency department after being involved in a high-speed motor vehicle collision. The patient was a restrained driver traveling on the highway when he suddenly slammed into the back of a stopped semi-truck. Upon arrival to the emergency department, the patient reports severe substernal chest pain radiating to his left shoulder blade. His temperature is 37.0°C (98.6°F), pulse is 104/min, respirations are 18/min and blood pressure is 185/100 mmHg. Physical examination shows bruising over the anterior chest wall, but no crepitus is felt on palpation. Shoulder examination shows full passive and active range of motion bilaterally. A chest X-ray is obtained (see below): Reproduced from: Radiopaedia Which of the following is the most likely location of injury? A. Manubrium B. Aortic isthmus C. Distal descending aorta D. Clavicle E. Sinotubular junction

B Segmental thrombosing inflammation with sparing of the internal elastic lamina This patient's signs of digit ulceration, superficial thrombophlebitis (painful nodules), and Raynaud phenomenon (vasospasm of the fingers in response to cold exposure), together with his significant smoking history, are suggestive of thromboangiitis obliterans (TAO). Segmental thrombosing inflammation of small and medium-sized vessels that spares the internal elastic lamina is the characteristic histologic finding in thromboangiitis obliterans. TAO most commonly occurs in male smokers < 45 years of age and manifests with migratory thrombophlebitis, Raynaud phenomenon, and digital ischemia with intermittent claudication. Eventually, critical limb ischemia may develop, which can result in rest pain of the distal extremities, peripheral pulselessness, and/or gangrene with autoamputation of digits.

A 38-year-old man comes to the physician because of a 3-week history of a painful rash affecting his left foot. For the past 2 years, he has had recurrent episodes of color changes in his fingers when exposed to the cold; his fingers first turn white and then progress to blue and red before spontaneously resolving. He has smoked two packs of cigarettes daily for 20 years. His blood pressure is 115/78 mm Hg. Physical examination shows multiple tender, dark purple nodules on the lateral surface of the left foot with surrounding erythema that follow the course of the lateral marginal vein. There are dry ulcers on the tip of his right index finger and on the distal aspect of his right hallux. Serum lipid studies show no abnormalities. Biopsy of the dorsalis pedis artery will most likely show which of the following findings? A Granulomatous inflammation with narrowing of the vessel lumen B Segmental thrombosing inflammation with sparing of the internal elastic lamina C Calcification of the tunica media with foam cells and fibrous cap formation D Intraluminal fibrin clot predominantly composed of red blood cells E Transmural inflammation with fibrinoid necrosis of the vessel wall

D. Segmental vasculitis extending to adjacent veins and nerves Major takeaway Thromboangiitis obliterans is a segmental, nonatherosclerotic inflammatory vasculitis that mainly affects small to medium-sized arteries, veins and nerves of the extremities. Patients are often young individuals with a smoking history who present with signs of distal ischemia. Main explanation This patient with a heavy smoking history, skin nodules, and finger pain and ulceration most likely has thromboangiitis obliterans (previously known as Buerger disease). Thromboangiitis obliterans (TAO) is a nonatherosclerotic, segmental inflammatory vasculitis that commonly affects small to medium-sized arteries, veins and nerves of the extremities. Patients are often young individuals with a smoking history who present with distal ischemia, ischemic digit ulcers, and digit gangrene. The disease is strongly associated with smoking, and smoking cessation is essential to decrease the risk for amputation. Although the pathophysiology of the disease is still not clear, the condition is distinguished from other forms of vasculitis by a highly cellular, inflammatory luminal thrombus with relative sparing of the vessel wall. In contrast to atherosclerosis, which involves the intima and media, thromboangiitis obliterans is characterized by the infiltration of inflammatory cells in all 3 layers of the arterial wall.

A 42-year-old man comes to the physician's office due to progressive finger pain and ulceration for the past 3 weeks. He describes sharp pain at the fingertips followed by dark color changes and ulceration. His medical history is unremarkable. The patient reports smoking 2 packs of cigarettes per day for the past 10 years. He does not use illicit drugs. The patient is from Israel. Vital signs are within normal limits. On cardiac auscultation, no murmurs are heard, but peripheral pulses are diminished. Examination of bilateral hands reveals superficial tender nodules parallel to the radial vein. Examination of the blood glucose level, hemoglobin A1C, and lipid panel are within normal limits. Which of the following pathologic findings is most likely to be seen in this patient? A. Focal granulomatous inflammation of the media B. Transmural inflammation of the arterial wall with fibrinoid necrosis C. "Onion-skin" thickening of the arterial wall D. Segmental vasculitis extending to adjacent veins and nerves E. Granulomatous, necrotizing vasculitis with eosinophilia F. Calcification of the media G. Fibrous-fatty intimal plaque

B. Non-ST-elevation myocardial infarction Objective Diagnose non-ST- elevation myocardial infarction . Major takeaway Non-ST- elevation myocardial infarction (NSTEMI) is characterized by chest pain at rest, the absence of ST-elevation on the ECG, and elevated biomarkers of myocardial injury. Main explanation The most likely diagnosis is a non-ST- elevation myocardial infarction (NSTEMI). This patient has had chest pain at rest, an elevated serum troponin I level, and an electrocardiogram that shows ST-segment depression that is particularly prominent in leads V2-V6. These features indicate a NSTEMI. In an acute coronary syndrome , obstruction to coronary blood flow results in either transient or prolonged episodes of severe myocardial ischemia. ST-elevation myocardial infarction (STEMI) is diagnosed in patients with a clinical presentation consistent with acute myocardial infarction together with electrocardiographic evidence of ST-segment elevation. Although most patients with STEMI ultimately develop Q waves on electrocardiogram, some exhibit diagnostic ST-segment elevation and cardiac enzyme elevations without Q-waves. Patients who are characterized by ischemic chest pain but without diagnostic ST-segment elevation are categorized as having unstable angina or NSTEMI. These two conditions differ in the severity of the myocardial ischemia. In NSTEMI, ischemia is severe and results in a detectable release of biomarkers of myocardial injury hours after the onset of ischemic chest pain, most commonly cardiac troponin I, troponin T, and the MB isoenzyme of creatine kinase. In unstable angina, there is no detectable increase in these enzymes. In non-ST- elevation myocardial infarction ( NSTEMI ), ischemia is severe and results in a detectable release of biomarkers of myocardial injury hours after the onset of ischemic chest pain, most commonly cardiac troponin I, troponin T, and the MB isoenzyme of creatine kinase. In unstable angina , there is no detectable enzyme increase.

A 66-year-old man comes to the emergency department because of left-sided chest pain that began at rest and lasted for 15 minutes before resolving. He says a similar episode occurred at rest yesterday. Medical history includes hypertension and type II diabetes mellitus. Current medications are amlodipine, glyburide, and aspirin. His temperature is 36.8°C (98°F), pulse is 87/min, respirations are 18/min, and blood pressure is 117/78 mm Hg. Cardiac examination shows carotid upstroke is normal, there are no cardiac murmurs, and the lung fields are clear. ECG shows ST-depression in leads V2, V3, and V4. Laboratory investigations show elevated CK-MB and serum troponin I level. Chest X-ray is normal. Which of the following is the most likely diagnosis? A. Chronic stable angina B. Non-ST-elevation myocardial infarction C. Pulmonary embolism D. ST-elevation myocardial infarction E. Unstable angina

E. Constrictive pericarditis Major takeaway Constrictive pericarditis is a condition characterized by incomplete filling of the cardiac chambers due to thickening of the parietal pericardium. Patients often present with fatigue, hepatomegaly, dyspnea, because the pericardium can not accommodate changes in cardiac volume. Main explanation The patient in the vignette is experiencing constrictive pericarditis , a condition characterized by incomplete filling of the cardiac chambers due to thickening of the parietal pericardium. When this happens, the abnormal pericardium can't stretch to accommodate physiologic changes in cardiac volume. Therefore, patients often present with a pericardial knock due to the ventricles hitting the thickened wall, while some others can have an absent apical impulse. Similarly, in the jugular venous pressure, the y descent, which usually corresponds to the right atrium emptying into right ventricle is going to be prominent.Tuberculosis is the most common cause of constrictive pericarditis worldwide, however, in some cases, scarring or calcification of the pericardium can occur due to uremic states (i.e., chronic renal failure), or previous open heart surgery. A chest radiograph usually shows dystrophic calcification. The definitive treatment for constrictive pericarditis is pericardial stripping, which is a surgical procedure where the entire pericardium is peeled away from the heart.

A 72-year-old woman comes to the office because of fatigue and progressive dyspnea despite optimal medical treatment for heart failure. Her medical history is relevant to heart failure, active tuberculosis, and chronic renal failure to which she has been in long-term hemodialysis for 12 years. She currently takes rifampin and isoniazid. Her physical exam is remarkable for the presence of hepatomegaly, jugular venous distention that fails to subside on inspiration, and an impalpable apical impulse. Her temperature is 36°C (97.4°F), pulse is 122/min, respirations are 16/min, and blood pressure is 120/60 mmHg. Her cardiac monitor shows a prominent y descent in her jugular venous pulse. A cardiac ultrasound shows pericardial calcifications and small tubular-shaped ventricles. Which of the following is the most likely cause of this patient's current condition? A. Dilated cardiomyopathy B. Hypertrophic cardiomyopathy C. Atrial Fibrillation D. Restrictive cardiomyopathy E. Constrictive pericarditis

A Coronary artery aneurysm The patient presents with signs and symptoms of Kawasaki disease, an acute vasculitis that primarily affects young children and is characterized by a high fever, desquamative rash, conjunctivitis, mucositis, cervical lymphadenopathy, and erythema and edema of the distal extremities. Coronary artery aneurysm is the most common complication of Kawasaki disease. Coronary artery aneurysms occur due to the acute necrotizing vasculitic involvement of the coronary arteries and can lead to myocarditis, pericarditis, arrhythmias, and myocardial infarction, the main cause of death in Kawasaki disease. Other cardiac sequelae such as heart failure due to decreased myocardial contractility, valvular regurgitation, and pericardial effusion can also occur

A previously healthy 3-year-old boy is brought to the physician by his parents because of fever and a rash for 6 days. His temperature is 38.9°C (102°F). Examination shows right-sided anterior cervical lymphadenopathy, bilateral conjunctival injection, erythema of the tongue and lips, and a maculopapular rash involving the hands, feet, perineum, and trunk. Which of the following is the most common complication of this patient's condition? A Coronary artery aneurysm B Rapidly progressive glomerulonephritis C Hearing loss D Retinopathy E Peripheral neuropathy

B. Takayasu arteritis Major takeaway Takayasu arteritis is a large-vessel vasculitis that mainly affects women at ages 10-40 years. One of the classic signs is weak peripheral pulses due to narrowing of the arteries from inflammation, as well as blood pressure discrepancy between affected limbs. Main explanation Takayasu arteritis (TA) is a large-vessel vasculitis that mainly affects the aorta and its branches. It is most often seen in women at ages 10-40 years, especially of Asian descent. Patients initially present with constitutional symptoms (e.g., night sweats, fatigue), followed by symptoms associated with vascular damage. The pathogenesis is poorly understood but appears to occur in a similar pathogenetic mechanism to temporal arteritis. TA is characterized as an inflammatory granulomatous vasculitis of medium- and large-sized arteries, which leads to transmural fibrous thickening of the arterial walls, resulting in multiple vascular obstructions and eventual ischemic changes. Arterial wall inflammation is characterized by a predominance of CD4+ T lymphocytes and macrophages, which undergo granulomatous organization and formation of giant cells. The inflammation induces vascular remodeling that leads to intimal hyperplasia and lumen occlusion. The following features may be seen: Constitutional symptoms: low-grade fever, weight loss, fatigue Cardiovascular: Absent or weak peripheral pulses, limb claudication, blood pressure discrepancy between arms, hypertension, angina, myocardial infarction Musculoskeletal: arthralgias, myalgias Dermatological: skin lesions similar to erythema nodosum or pyoderma gangrenosum Neurologic symptoms, caused by involvement of the carotid and vertebral arteries: lightheadedness, vertigo, syncope, headaches, and strokes Laboratory findings: elevated ESR, CRP, anemia of chronic disease, thrombocytosis and leukocytosis Since TA is primarily an autoimmune condition, initial treatment of symptomatic TA begins with corticosteroids. In more severe conditions, surgical revascularization is a possible therapy as well.

A previously healthy Japanese 36-year-old woman comes to the clinic due to headaches, lightheadedness, diffuse arthralgia, and numbness in the hands for the past week. She also reports fatigue and waking up in the middle of the night "soaked in sweat." She does not smoke cigarettes or use illicit drugs. Temperature is 36.5°C (97.7°F), and blood pressure is 140/95 mmHg in the left arm and 110/65 in the right arm. Physical examination reveals a weak right radial pulse. A systolic bruit is heard over the right subclavian artery. Laboratory results are obtained and shown below: A. Thromboangiitis obliterans B. Takayasu arteritis C. Polyarteritis nodosa D. Granulomatosis with polyangiitis E. Microscopic polyangiitis F. Temporal arteritis

E. Vision loss Major takeaway Giant cell arteritis (GCA) can lead to permanent vision loss. Patients with suspected GCA should be treated empirically with glucocorticoids, even before diagnostic evaluation by temporal artery biopsy is complete. Main explanation This patient's throbbing unilateral headache combined with the finding of a pulsating temporal artery is strongly suggestive of giant cell (temporal) arteritis (GCA). GCA is a vasculitis of medium and large vessels, typically affecting individuals over 50 years of age. In addition to headache, suggestive signs of GCA are jaw claudication and systemic symptoms (fever, fatigue, weight loss). The most feared complication of GCA is permanent vision loss, typically due to anterior ischemic optic neuropathy (AION) secondary to occlusion of the posterior ciliary artery. Consequently, physicians should maintain a high clinical suspicion for GCA and initiate glucocorticoid treatment as soon as the diagnosis is suspected, even before it is confirmed. Temporal artery biopsy should be performed as soon as possible to confirm the diagnosis.

An 85-year-old woman comes to the emergency department because of a throbbing right-sided headache for the past 3 hours. She describes exquisite tenderness over her right temple without associated nausea or photophobia. Physical examination shows a pulsating right temporal artery. Which of the following conditions is the patient at an increased risk for developing? A. Brain metastasis B. Hearing loss C. Stroke D. Subarachnoid hemorrhage E. Vision loss

B. Atherosclerosis Major takeaway Transient vision loss, also known as amaurosis fugax , is the subjective phenomenon caused by a transient and temporary ceasing of the retinal blood flow, most likely due to atherosclerosis of the carotid arteries. Main explanation Atherosclerosis of the carotid arteries is the most common cause associated with amaurosis fugax. Amaurosis fugax is an all-inclusive term for all forms of transient visual loss, with "amaurosis" meaning dark, and "fugax" translating to fleeting. The causes of this vision loss can be roughly classed as embolic, hemodynamic, ocular, neurologic, or idiopathic. Atherosclerosis can lead to ischemia via both narrowing of carotid arteries as well as predisposition to embolic disease. Carotid duplex ultrasonography or an MRA (magnetic resonance angiography) should be performed to assess the degree of carotid blockage. Funduscopic examination may show signs of vascular damage including plaques.

An 85-year-old woman comes to the emergency department due to temporary loss of vision in her left eye. History reveals the patient had a five minute episode of visual loss and describes it as if a "shade" was pulled over her left eye. Past medical history reveals hypertension and diabetes. Neurologic and ophthalmologic examinations are normal. Which of the following is the most likely cause of this condition? A. Long history of smoking B. Atherosclerosis C. Alcohol D. Diabetes E. Hypertension

E. Myeloperoxidase enzyme Major takeaway Eosinophilic granulomatosis with polyangiitis (Churg-Strauss) is an autoimmune small-vessel vasculitis characterized by asthma and marked eosinophilia. Patients are often found positive for anti-myeloperoxidase (MPO-ANCA), a lysosomal granule constituent which is involved in oxygen free radical generation. Main explanation This young patient with a history significant for chronic asthma , recurrent sinusitis, and nasal polyposis, now presents with new skin lesions and marked eosinophilia, and therefore, he most likely has eosinophilic granulomatosis with polyangiitis (Churg-Strauss). Patients with vasculitis have circulating antibodies that react with neutrophil cytoplasmic antigens termed antineutrophil cytoplasmic antibodies (ANCAs). ANCAs are a heterogeneous group of autoantibodies directed against neutrophil primary granules elements (mainly enzymes). ANCAs are useful diagnostic markers, and their titers mirror clinical severity and predictor of disease recurrence. ANCAs are classified accordintosis with polyangiitis (Wegener granulomatosis).

A 20-year-old man comes to the clinic due to the recent appearance of skin lesions. The patient's history is significant for chronic asthma, recurrent sinusitis, and nasal polyposis. On physical examination, tender subcutaneous nodules are present on the bilateral shins. The results of the white blood cell differential are shown below: This patient's condition is associated with antibodies directed against which of the following? A. Transglutaminase enzyme B. Basement membrane C. Proteinase D. Mitochondria E. Myeloperoxidase enzyme F. Microsome

(B) Colchicine Answer B (correct answer):This man has pericarditis, inflammation of the pericardial lining of the heart. Most cases are autoimmune with an unknown trigger, although other causes include viral infection, rheumatologic disorders (lupus, rheumatoid arthritis) and post myocardial infarction. Pericarditis is largely a benign condition but may mimic more serious disorders such as myocardial ischemia. The diagnosis is based on clinical signs and symptoms plus supportive ECG findings - there are no diagnostic imaging or laboratory tests. The pain of pericarditis is usually present at rest. It worsens with lying flat and improves with sitting upright. The pain is often worse with inspiration (pleuritic). Cardiac exam may reveal a friction rub. ECG may show PR depressions and ST elevations involving multiple different anatomic territories of the left ventricle. Treatment of pericarditis is with non-steroidal anti-inflammatory drugs (NSAIDs), glucocorticoids or colchicine. Colchicine inhibits white blood cells. It has been shown to improve symptoms in pericarditis when used with NSAIDs. Glucocorticoids are often reserved for patients who cannot take NSAIDs. Answer A (incorrect answer): Tacrolimus is a calcineurin inhibitor. It is a potent immunosuppressant used to prevent transplant rejection and in some autoimmune disorders. Answer C (incorrect answer): Lorazepam is a benzodiazepine. It has no role in treating pericarditis. Answer D (incorrect answer): Acetaminophen is a weak analgesic that inhibits cyclooxygenase mostly in the CNS reducing pain. It is inactive peripherally and does not suppress inflammation. Answer E (incorrect answer): Cyclophosphamide is an alkylating agent. It is a potent immunosuppressant used to prevent transplant rejection and in some autoimmune disorders.

A 22-year-old man is evaluated for chest pain. He has chest pain at rest that is sharp and improves with sitting forward. On cardiac exam there is a friction rub. Which of the following medications is the best therapy? (A) Tacrolimus (B) Colchicine (C) Lorazepam (D) Acetaminophen (E) Cyclophosphamide

E. Pulmonary embolism Major takeaway Pulmonary embolism should be suspected in young female patients with new onset shortness of breath and a history of obesity, smoking, and oral contraceptive use. Main explanation Pulmonary embolism is caused by an embolus (most commonly from deep venous thrombosis formation in the lower extremities) which travels to and lodges in the pulmonary vasculature. Symptoms of pulmonary embolism can be non-specific and include shortness of breath, chest pain which worsens with deep inspiration, a swollen and erythematous calf, tachycardia, tachypnea, fever, and hemoptysis. Risk factors for pulmonary embolism include a family history of hypercoagulability, immobilization, recent surgery, and malignancy. Although less common, pulmonary embolism can also occur in young women especially in the setting of obesity, tobacco, and oral contraceptive use. The classic changes seen on electrocardiogram are a prominent S-wave in lead I, Q-wave in lead III,and inverted T wave in lead III. A massive and acute pulmonary embolism can lead to right axis deviation and a new right bundle branch block. However, an electrocardiogram is neither sensitive nor specific for the diagnosis of pulmonary embolism. Hence, diagnosis in patients with high probability of pulmonary embolism is done with a spiral computed tomography scan. Other imaging modalities such as chest X-rays (which may show a Westermark sign or Hampton hump) and echocardiography (showing a McConnell sign) may be utilized to support the diagnosis.

A 27-year-old woman comes to the emergency department because of shortness of breath for the past hour. She says that she has been studying for her nursing licensing examination and has spent most of the past month studying in the library. She has a history of obesity and menorrhagia controlled with an oral contraceptive pill. She smokes a pack a day for the past 10 years. Her temperature is 36.4°C (97.5°F), pulse is 110/min, respirations are 24/min, and blood pressure is 135/85 mm Hg. The lungs are clear to auscultation. An electrocardiogram is done showing a prominent S-wave in lead I, a Q-wave in lead III, and an inverted T-wave in lead III. Which of the following is the most likely diagnosis? A. Atrial flutter B. Chronic obstructive pulmonary disease exacerbation C. Community acquired pneumonia D. Iron deficiency anemia E. Pulmonary embolism

F. Mitral regurgitation Major takeaway Kawasaki disease is a medium-vessel vasculitis seen in children that may lead to cardiac involvement. During the acute phase, complications such as coronary artery aneurysm and mitral regurgitation may occur if left untreated. Main explanation This child presenting with a high fever for more than 5 days, possible mucositis (pain during feeding), cervical lymphadenopathy, polymorphous desquamating rash, and conjunctival injection most likely has Kawasaki disease. Kawasaki disease, also known as mucocutaneous lymph node syndrome is a medium-vessel vasculitis seen in children under 5 years old. The diagnostic criteria for Kawasaki disease is in the chart below.A) Bilateral, non-exudative conjunctival injection with perilimbal sparing. (B) Strawberry tongue and bright red, swollen lips with vertical cracking and bleeding. (C) Erythematous rash involving perineum. (D) Erythema of the palms, which is often accompanied by painful edema of the dorsa of the hands. (E) Erythema of the soles, and swelling dorsa of the feet. (F) Desquamation of the fingers. (G) Erythema and induration at the site of a previous vaccination with Bacille-Calmette-Guérin (BCG). (H) Perianal erythematous desquamation. Reproduced by: Wikimedia CommonsThe main complication seen in Kawasaki disease is coronary artery abnormalities, which may include dilation, aneurysm, and/or stenosis. During the acute phase, patients may develop depressed ventricular function, valvular regurgitation, and pericardial effusion. Late complications consist of myocardial infarction, arrhythmias, stenosis, and occlusion. Acute myocardial infarction is the main cause of death in Kawasaki disease, and early treatment with intravenous immunoglobulin (IVIG) and aspirin is important to avoid this complication. Kawasaki disease is one of the few cases where it is acceptable to treat children with aspirin (generally aspirin is avoided in children due to the risk of developing Reye syndrome).

A 3-year-old boy is brought to the emergency department by his parent due to a fever for the past six days. The fever has not responded to treatment with acetaminophen. His parent mentions that the patient has refused to eat due to pain in his mouth. During his bath yesterday, the parent also noticed that he has a rash covering his trunk and limbs. Temperature is 39°C (102.2°F), pulse is 125/min, and blood pressure is 90/55 mm Hg. Examination shows an irritable-appearing child with a polymorphous rash covering the trunk which peels off near the fingers, as well as bilateral eye injection without exudate. He has oral erythema. Cervical lymphadenopathy is present. Which of the following complications may be seen in this patient if left untreated? A. Reye syndrome B. Subacute sclerosing panencephalitis C. Staphylococcal scalded skin syndrome D. Hemorrhagic cystitis E. Glomerulonephritis F. Mitral regurgitation

A. Thoracic aortic aneurysm Major takeaway Thoracic aortic aneurysms are true aneurysms that lead to greater than 50% dilation compared to a normal aorta. Risk factors include collagen/elastin disorders, such as Ehlers-Danlos syndrome and Marfan syndrome, as well as bicuspid aortic valves, hypertension, and smoking. Most cases are silent, but if left untreated can progress to aortic dissection and rupture. Main explanation This patient has features of Marfan syndrome, including pectus excavatum (concave anterior chest wall) and mitral valve prolapse, given the mid-systolic murmur at the cardiac apex on examination. Both of these features, in conjunction with a history of smoking and dull chest pain radiating to the back, are concerning features for a diagnosis of a thoracic aortic aneurysm (TAA). TAAs are true aneurysms, meaning all three layers of the blood vessel - intima, media, and adventitia - are dilated >50% greater of the normal diameter of the aorta. Other risk factors for thoracic aortic aneurysms include hypertension and hypercholesterolemia. Significant genetic risk factors aside from Marfan syndrome include other inherited connective tissue disorders such as Loeys-Dietz and vascular type Ehlers-Danlos syndromes, large vessel inflammatory diseases such as giant cell arteritis and Takayasu's aortitis, as well as infections such as tertiary syphilis. Most cases of TAAs are silent, meaning they do not produce symptoms until they are large enough to compress on adjacent structures. The symptoms of TAAs depend largely on their location: Aneurysms of the aortic arch result in ascending aneurysms and can present signs of aortic regurgitation due to dilation of the aortic valve annulus; this can eventually progress to heart failure. In addition, symptoms can be related to compression of nearby structures: for example, the coronary arteries (chest pain) as seen in this patient, the tracheobronchial tree (wheezing) or the esophagus (dysphagia). Aneurysms of the descending aorta are less likely to lead to symptoms until the aneurysm has become very large due to the capacity of the thoracic cavity. If large enough, the aneurysm can affect the adjacent spine, causing back pain. Incorrect: While people with marfanoid habitus and thoracic aortic aneurysms are at risk for the development of aortic dissection , the clinical presentation is usually much more severe. Patients are often extremely uncomfortable, hypertensive , and describe a sensation of ripping or tearing chest pain radiating to the back.

A 35-year-old man comes to the emergency department with a one day history of dull chest pain. The patient reports he was out for a jog yesterday when he suddenly experienced dull chest pain radiating to the back, with associated shortness of breath. The patient reports he is otherwise healthy but has smoked 5-10 cigarettes per day for the past ten years. His temperature is 37.0°C (98.6°F), pulse is 78/min, respirations are 18/min, blood pressure is 130/66 mmHg, and O2 saturation is 99% on room air. He does not appear to be in acute distress. Physical examination shows a thin, tall man, with a concave anterior chest wall. Cardiovascular examination shows 2+ pulses in the upper extremities bilaterally, and a mid-systolic murmur at the cardiac apex is heard on auscultation. Which of the following is the most likely diagnosis? A. Thoracic aortic aneurysm B. Costochondritis C. Pulmonary embolism D. Aortic dissection E. Coronary artery disease

B. Coronary artery aneurysm Major takeaway Kawasaki disease is a medium-vessel arteritis that resembles a viral disease in its presentation. Coronary artery aneurysm (along with aneurysms in other locations) is a complication of untreated Kawasaki disease. Main explanation Kawasaki disease is a medium-vessel vasculitis seen in children under 5 years old. Diagnostic criteria include at least 5 days of fever, erythema of oral cavity or cracked lips, a rash on the trunk, erythematous swelling or skin peeling on hands and feet, nonsuppurative conjunctivitis and swollen lymph nodes in the neck.Coronary artery aneurysms occur in 20-25% of untreated children, usually detected after around 10 days of fever, although they may take several weeks to develop. In developed nations, it appears to have replaced acute rheumatic fever as the most common cause of acquired heart disease in children. Death can occur due to myocardial infarction after thrombosis or rupture of a large coronary aneurysm. Early treatment with intravenous immunoglobulin (IVIG) and aspirin is important to avoid this complication. Kawasaki disease is one of the few cases where it is acceptable to treat children with aspirin (generally aspirin is avoided in children because of the risk of developing Reye's syndrome).Other complications of Kawasaki disease may include valvular insufficiencies, aneurysms in other locations, intestinal obstruction or ischemia, and ophthalmologic changes including uveitis, amaurosis, and iritis.

A 3-year-old child is brought to the emergency department by his mother because of a 6-day high fever. The fever has not responded to treatment with paracetamol. His temperature is 39°C (102.2°F). Examination shows an irritable appearing child with cracked lips and peeling skin on his fingertips. There is also injected conjunctiva without discharge, a swollen tongue, swollen cervical lymph nodes, and a maculopapular rash on his trunk. Which of the following complications is this child most likely at risk of if prompt treatment is not initiated? A. Acute renal failure B. Coronary artery aneurysm C. Monocular blindness D. Reye syndrome

D. Pericarditis Major takeaway Pericarditis can be differentiated from myocardial infarction and pulmonary embolism by the quality of the chest pain (sharp, no worsening with exertion, alleviation with sitting up) and the nature of ECG changes (diffuse ST-elevation and/or PR-depression). Main explanation Pericarditis refers to inflammation of the pericardium, a double layer of tissue that surrounds the heart. Causes of primary pericarditis are often pathogenic in nature, with viruses as the most common culprit. Pericarditis more commonly occurs secondary to other systemic diseases, with the most common being uremia. Rheumatic fever, systemic lupus erythematosus, myocardial infarction, and metastatic cancer are other etiologies for secondary pericarditis. Pericarditis is characterized by chest pain unrelated to exertion and worse when laying down. A friction rub may be heard on physical examination. ECG changes in pericarditis include diffuse ST-segment elevation and/or PR-segment depression. Histologic changes depend on the type of pericarditis. Acute viral infection or uremia leads to a fibrinous pericarditis, with a shaggy appearance to the pericardium. Acute bacterial pericarditis causes a fibrinopurulent pericarditis with the presence of pus. Acute pericarditis usually resolves on its own. However inflammation may cause significant pericardial effusion leading to cardiac tamponade. Additionally, extensive disease can lead to fibrosis, resulting in a chronic, constrictive pericarditis. Management of pericarditis consists of non-steroidal anti-inflammatory drugs (NSAIDs). Colchicine is also recommended in a first episode of acute pericarditis to minimize symptoms. Glucocorticoids are used if disease is refractory to the previous agents.

A 30-year-old woman comes to the emergency department because of sharp chest pain for one day. She first felt the pain as she was sitting on her couch watching television. The pain is in the center of her chest and is worse with deep inspiration and coughing. She reports that her pain improves when she leans forward while sitting. An ECG is obtained and the results are shown below. Which of the following is the most likely diagnosis? A. Aortic dissection B. Myocardial infarction C. Pancreatitis D. Pericarditis E. Pulmonary embolism

A. Diabetes mellitus High carbohydrate intake may cause increased levels of atherosclerosis ; however, it is not classed as a major risk factor and has yet to have a direct causal link proven. Major takeaway Atherosclerosis can be caused by diabetes mellitus, especially in younger patients. Diabetes complications can be grouped into macrovascular (ischemic heart disease, cerebrovascular disease, and peripheral vascular disease) and microvascular (neuropathy, retinopathy, or nephropathy) complications. Main explanation This patient has multiple risk factors for atherosclerosis , and as a result has developed ischemic heart disease, specifically stable angina. Of the risk factors in the question, diabetes mellitus is the most significant. It is a proven major risk factor for the formation of atherosclerotic plaques, and can be a cause of extensive vascular disease, even in young people. Most of the pathologies associated with diabetes mellitus are a direct result of this vascular disease. When large vessels are affected it results in ischemic heart disease (e.g. heart attack), cerebrovascular disease (e.g. stroke), and peripheral vascular disease (e.g. claudication or arterial ulcers). When small vessels are affected, patients may develop neuropathy (nerve disease), retinopathy (diabetic eye disease), or nephropathy (chronic kidney failure).The mechanism by which diabetes is thought to induce atherosclerotic disease is through hyperglycemic damage to the intimal layer of arteries.

A 31-year-old man comes to the office because of chest pain with activity for the past 6 months. He is a businessman and he says his job is stressful, but has not been more stressful than usual. He has diabetes mellitus. Examination shows a BMI of 28.5 kg/m2 and blood pressure of 137/83 mm Hg. Coronary angiogram shows 75% narrowing of the left anterior descending coronary artery. Which of the following risk factors is most significant for this patient? A. Diabetes mellitus B. High carbohydrate intake C. Lack of exercise D. Obesity E. Stress

(E) Cocaine abuse Answer E (correct answer): This patient has signs and symptoms of aortic dissection. This condition is caused by a tear in the intimal layer of the aorta which allows blood to enter the space between the intima and the media. When this occurs, blood can "dissect" (i.e. separate) the two layers and disrupt blood flow through the aorta and into large arteries. Classic clinical findings are chest pain radiating to the back, widened mediastinum on imaging and blood pressure differential between the left and right arms. The most common predisposing factor leading to aortic dissection is hypertension. Most patients with hypertension-related dissection are older with a long-standing elevation of blood pressure that leads to aortic damage. This patient is very young for hypertension-related aortic dissection. He most likely has some predisposing condition other than primary hypertension. The two classic conditions that lead to aortic dissection in a young patient are Marfan syndrome and bicuspid aortic valve. In addition, cocaine abuse is also associated with aortic dissection. Cocaine blocks the re-uptake of norepinephrine. This increases the activity of the sympathetic nervous system. This causes an abrupt rise in blood pressure that may lead to aortic dissection. Strategy tip: If you did not know cocaine abuse is associated with aortic dissection you can still identify the correct answer by knowing that hypertension leads to dissection. Of the answer choices, only cocaine leads to elevated aortic pressure. Answer B (incorrect answer): Hypertrophic cardiomyopathy is associated with sudden cardiac death from arrhythmias. Answer C (incorrect answer): Mitral stenosis leads to elevated left atrial pressure. The left atrium may become massively dilated and compress the esophagus or recurrent laryngeal nerve. Answer D (incorrect answer): Nonischemic cardiomyopathy leads to heart failure signs and symptoms. Answer A (incorrect answer): Primary pulmonary hypertension causes dyspnea and right heart failure but not aortic dissection.

A 31-year-old man presents for evaluation of chest pain. One hour ago, he noted the abrupt onset of chest pain that radiates to his back. Blood pressure is 162/92 mmHg and pulse is 110/min. Pulses are diminished in the left arm. A 2/6 diastolic murmur is heard at the left sternal border. Chest X-ray shows widening of the aorta. Which of the following conditions may have led to this man's presentation? (A) Primary pulmonary hypertension (B) Hypertrophic cardiomyopathy (C) Mitral stenosis (D) Nonischemic cardiomyopathy (E) Cocaine abuse

Major takeaway Pericardial effusions can present with exertional dyspnea, pulmonary edema, and jugular venous distention. The diagnosis can be confirmed with transthoracic echocardiography, a noninvasive modality that can also be used to guide therapy. Main explanation This patient is most likely suffering from a gradually developing pericardial effusion causing cardiac tamponade. Pericardial effusion can be associated with a variety of etiologies, including autoimmune disease and malignancy. Accumulation of fluid in the pericardial sac restricts ventricular filling during diastole, leading to signs of left-sided heart failure. These signs include exertional dyspnea and pulmonary edema, manifested as bibasilar crackles. Right-sided heart failure manifests as pulsus paradoxus and jugular venous distention. Pericardial effusion also causes characteristic signs on ECG, including sinus tachycardia, low QRS voltage, and electrical alternans. In patients with suspected pericardial effusion, a transthoracic echocardiogram (TTE) should be performed to confirm the diagnosis. This noninvasive test is highly sensitive and specific for pericardial effusion and can also be used to guide therapy, using pericardiocentesis to drain the effusion. On chest x-ray , a pericardial effusion surrounding the heart gives the cardiac silhouette a characteristic "water bottle" shape. However, the presence of an effusion must be confirmed by another imaging modality.

A 32-year-old woman comes to the emergency department because of a 3-week history of progressive exertional dyspnea. She reports that she has to stop and rest after walking a single block. She has no dyspnea at rest and has otherwise felt well. She has a 5-year history of systemic lupus erythematosus. Her pulse is 105/min, respirations are 22/min, and blood pressure is 110/72 mm Hg. Cardiac examination shows distant heart sounds and jugular venous distention. Bibasilar crackles are heard. An ECG is shown. Which of the following is the most appropriate next step in diagnosis? A. Transthoracic echocardiography B. Chest x-ray C. Transesophageal echocardiography D. CT scan of the chest E. Sphygmomanometry

D Answer B (correct answer): This is a typical presentation of peri-infarction pericarditis (fibrinous pericarditis). Myocardial inflammation after myocardial infarction (MI) sometimes involves the pericardium leading to pericarditis. This occurs within the first few days following MI. The pain of pericarditis is usually present at rest. It worsens with lying flat and improves with sitting upright. The pain is often worse with inspiration (pleuritic). Cardiac exam may reveal a friction rub. Answer A (incorrect answer): Cardiac tamponade occurs when a large pericardial effusion limits ventricular filling. This patient has a small pericardial effusion - a common finding in pericarditis. In addition, she has no change in blood pressure with inspiration indicating absence of a pulsus paradoxus, the classic hallmark of tamponade. Answer C (incorrect answer): Systolic heart failure causes dyspnea, fatigue, and edema. Lungs will have rales. An S3 is often present. Jugular venous pressure is usually elevated. Answer D (incorrect answer): Pulmonary embolism may cause pleuritic chest pain as seen in this case. In addition, hospitalized patients on bed rest are at risk due to lack of mobility which causes stasis of blood. However, pulmonary embolism does not cause a friction rub. Answer E (correct answer): Dressler's syndrome is an autoimmune form of pericarditis. It can occur following MI but onset is 4 to 6 weeks later.

A 32-year-old woman is evaluated for dyspnea. She was recently diagnosed with Hodgkin's lymphoma. For the past two days she has had fatigue and exertional dyspnea. Exam reveals fine bibasilar rales. Heart sounds are distant with no murmurs. Jugular venous pressure is 12 mmHg (normal <8). Blood pressure is 100/70 mmHg and falls during inspiration. Echocardiography reveals a large pericardial effusion. She is treated with pericardiocentesis and removal of 1 liter of fluid from the pericardial space. Following this intervention, which of the following changes are expected for cardiac output and right ventricular end-diastolic pressure (RVEDP)? A B C D E F G H

(A) Elevated pericardial pressure Answer A (correct answer): This woman has a typical presentation of cardiac tamponade. This disorder occurs in the setting of a pericardial effusion when pericardial pressure rises to the point that ventricular filling is impaired. This leads to elevated diastolic pressures in the ventricles and atria. Acute tamponade may occur after trauma and presents with Beck's triad: distended neck veins, distant heart sounds, and hypotension. This patient has a subacute presentation that has developed over several days. This occurs when tamponade is caused by malignancy, uremia or following pericarditis. Typical findings of tamponade include elevated jugular venous pressure and distant heart sounds. A fall in systolic pressure during inspiration (pulsus paradoxus) is a classic finding. ECG may show sinus tachycardia, low voltage or rarely electrical alternans. Chest X-ray shows an enlarged cardiac silhouette. Echocardiography is the test of choice to demonstrate the presence of an effusion. Answer B (incorrect answer): Left atrial pressure is elevated in cardiac tamponade, not reduced. Right atrial pressure and right ventricular diastolic pressure are also elevated. Answers C, D (incorrect answers): Left ventricular contractility and right ventricular contractility are normal in cardiac tamponade. The disorder is caused by impaired filling with intact contractility. Answer E (incorrect answer): Right ventricular outflow tract obstruction is seen in Tetralogy of Fallot.

A 32-year-old woman is evaluated for dyspnea. She was recently diagnosed with Hodgkin's lymphoma. For the past two days, she has had fatigue and exertional dyspnea. Exam reveals fine bibasilar rales. On cardiac exam, heart sounds are distant with no murmurs. Jugular venous pressure is 12 mmHg (normal <8). Blood pressure is 100/70 mmHg and falls during inspiration. ECG shows low voltage and sinus tachycardia. Which of the following is the primary cause of this woman's presentation? (A) Elevated pericardial pressure (B) Reduced left atrial pressure (C) Reduced left ventricular contractility (D) Reduced right ventricular contractility (E) Right ventricular outflow obstruction

D. Postpartum early ambulation Major takeaway Early ambulation after delivery decreases the chance of a deep venous thrombosis and hence minimizes the risk for a pulmonary embolism. Main explanation Pulmonary embolism is a condition in which an embolus (most likely from an existing deep venous thrombosis or DVT) travels to the pulmonary vasculature. Symptoms of pulmonary embolism are fairly diverse and include shortness of breath, tachypnea, tachycardia, and a swollen, tender, and erythematous calf. Risk factors include a recent history of surgery and immobility. Pregnant and immediate post-partum patients are at an increased risk factor for DVT formation and pulmonary embolism. Hypercoagulability during pregnancy is a physiologic response in preventing massive post-partum hemorrhage. In addition, patients who undergo cesarean section are 3-5 times more likely to develop a DVT than a patient with a vaginal delivery. Because of this risk, patients who have had uncomplicated pregnancies and deliveries should be encouraged ambulate early after delivery. Certain patients at high risk for DVT formation can receive compression stockings and prophylactic low molecular weight heparin

A 33-year-old woman, gravida 2, para 2, in the postpartum unit is now having acute shortness of breath for the past 30 minutes. The patient gave birth to a healthy baby boy by elective repeat cesarean section 2 days ago. She says that she is still in extreme pain from the operation and has only left her bed a few times to use the restroom. Her temperature is 36.4°C (97.5°F), pulse is 107/min, respirations are 25/min, and blood pressure is 121/84 mm Hg. Physical examination shows clear lung fields bilaterally and a swollen, tender, and erythematous left calf. Which of the following would most likely have prevented this patient's complication? A. Appropriate pain control with morphine B. Incentive spirometer use C. Postpartum antibiotic administration D. Postpartum early ambulation E. Postpartum magnesium sulfate

D. Pulmonary artery Major takeaway Obstruction causes increased pressures along the path of flow, and decreased pressures distally. In tension pneumothorax , these changes take place in the pulmonary vasculature, and transmit to the heart, resulting in increased pressures in the pulmonary artery and right heart and decreased pressures in the pulmonary veins and left heart. Main explanation This patient is presenting with shock in the trauma setting, evidenced by hypotension and tachycardia. The differential diagnosis for this scenario includes tension pneumothorax, cardiac tamponade, and massive hemorrhage. Tension pneumothorax occurs as a result of trauma to chest wall that allows the entry of outside air into the interpleural space, a potential space with a negative pressure relative to that of the atmosphere. Trauma results in the equalization of pressures and consequently a collapse of the lung on the affected side, leading to absent breath sounds. This collapse creates an obstruction that increases resistance (and therefore pressure) proximally along the path of flow, and decreases volume (and therefore pressure) distally. The increased resistance impedes normal right-sided flow, and causes blood to back up into the systemic circulation, resulting in a dilation of the neck veins. Hepatomegaly may also be present. The volume of blood returning to the left side of the heart is decreased, resulting in low filling pressures of the pulmonary veins, left atrium, left ventricle, and aorta.

A 34-year-old man comes to the emergency department via ambulance because of a motor vehicle collision in which he was not wearing a seatbelt. On the way to the hospital he developed shortness of breath and tachycardia. He is in obvious discomfort, and his pulse is 120/min, respirations are 32/min, and a blood pressure is 80/50 mm Hg. He opens his eyes spontaneously, he can move all four extremities, and obeys commands. His neck veins are distended. Breathing sounds are absent on the right side. In which of the following structures is pressure most likely increased? A. Aorta B. Left atrium C. Left ventricle D. Pulmonary artery E. Pulmonary veins

E. Necrotizing vasculitis with systemic involvement Major takeaway Thromboangiitis obliterans is a vasculitis affecting the hands and feet of smokers, leading to claudication, ischemic pain, and possible gangrene and auto-amputation of affected digits. Treatment is smoking cessation. Main explanation This man with a chronic smoking history and claudication of his lower limbs has thromboangiitis obliterans, a segmental thrombotic vasculitis of the vessels of the hands and feet. This disease is more commonly seen in men than in women, and typically presents around 40 years of age. It is characterized by ischemic pain, secondary to the inflammatory thrombi in the arteries and vessels, as well as tender nodules in a venous distribution. As the ischemia progresses, patients will develop ulcers in the affected digits, accompanied by discoloration and eventual auto-amputation. Raynaud's phenomenon is another common symptom. This is typically a clinical diagnosis, though it may require the ruling out of similar conditions via biopsy. The only treatment for this condition is to quit smoking. Complete cessation should halt the progression of symptoms, and may even lead to remission. This sort of vascular inflammation is seen in granulomatosis with polyangiitis and microscopic polyangiitis , ANCA-associated vasculitides . The former typically involves the respiratory tract and involves granulomas while the latter commonly presents with glomerulonephritis and pulmonary capillaritis.

A 38-year-old man comes to urgent care for intermittent pain in his feet. The pain has been going on for several months, and he has recently noticed "tender bumps" on both feet. Pain is worse with prolonged walking, but improves with rest. His past medical history includes hypertension, for which he takes lisinopril. He works in carpentry shops, has smoked a pack a day for the past 20 years, and 4 alcoholic drinks per week. Which of the following best describes the cause of the patient's symptoms? A. Focal granulomatous inflammation B. Transmural arterial inflammation with necrosis C. Segmental arterial inflammation secondary to thrombi D. Eosinophilic granulomatous vasculitis E. Necrotizing vasculitis with systemic involvement

C-antineutrophil cytoplasmic autoantibody Major takeaway Granulomatosis with polyangiitis (GPA) is an antineutrophil cytoplasmic autoantibody (ANCA)-associated vasculitis which mainly affects small- and medium-sized vessels. The condition is associated with anti-proteinase 3 antibodies. Main explanation This patient with a history of fatigue , weight loss, and persistent bloody rhinorrhea is now presenting with shortness of breath and a productive bloody cough. Her presentation, in conjunction with the urinalysis results and unresponsiveness to antibiotic therapy, is consistent with granulomatosis with polyangiitis (GPA). GPA, formerly referred to as Wegener granulomatosis, is a systemic autoimmune disorder that belongs to the antineutrophil cytoplasmic autoantibody (ANCA)-associated vasculitides (together with microscopic polyangiitis and eosinophilic granulomatosis with polyangiitis). It mainly affects small- and medium-sized vessels throughout the body. Classic signs and symptoms are involvement of 3 systems: the upper respiratory tract (e.g. saddle nose deformity, nasal mucosal ulceration, otitis media), lower respiratory tract (e.g.. hemoptysis, lung nodules), and the kidneys (glomerulonephritis, hematuria). Lung involvement that does not resolve with antibiotic therapy should raise a suspicion of a non-infectious etiology. Patients with vasculitis have circulating antibodies that react with neutrophil cytoplasmic antigens termed "antineutrophil cytoplasmic antibodies" (ANCAs). ANCAs are a heterogeneous group of autoantibodies directed against neutrophil primary granules elements (mainly enzymes). Anti-proteinase-3 (PR3-ANCA), previously called c-ANCA, is a neutrophil azurophilic granule component that shares similar structure with numerous microbial peptides, possibly explaining the generation of these antibodies. PR3-ANCAs are associated with

A 38-year-old woman comes to the emergency department because of shortness of breath and a cough productive of blood-tinged sputum for 3 days. She had a similar episode several weeks ago and was prescribed antibiotics by her family doctor with minimal improvement. Over the past 2 months, she has experienced mild fever, malaise, and weight loss as well as persistent red-tinged rhinorrhea. She does not smoke cigarettes or use illicit drugs. Temperature is 38.0°C (100.4°F), pulse is 92/min, respirations are 18/min, and blood pressure is 110/68 mm Hg. Physical examination shows nasal crusting and multiple ulcer formations in the nasal cavity. Laboratory studies show an elevated erythrocyte sedimentation rate (ESR). Urinalysis shows dysmorphic red cells and red blood cell casts. Which of the following antibodies is most likely to be found in the patient's serum? A. Anti-myeloperoxidase antibodies B. Anti-centromere antibodies C. Anti-mitochondrial antibodies D. Anti-dsDNA antibodies E. C-antineutrophil cytoplasmic autoantibody

C. Anti-proteinase-3 antibodies Major takeaway Granulomatosis with polyangiitis (GPA), formerly referred to as Wegener's granulomatosis (WG), is a systemic disorder that involves both granulomatosis and polyangiitis. It is associated with cytoplasmic anti-neutrophil cytoplasmic antibodies (c-ANCA), and anti-proteinase 3 antibodies. Main explanation Granulomatosis with polyangiitis (GPA), formerly referred to as Wegener granulomatosis (WG), is a systemic disorder that involves both granulomatosis and polyangiitis. It is a form of vasculitis (inflammation of blood vessels) that affects small- and medium-sized vessels in many organs. Damage to the lungs and kidneys can be fatal and it requires long-term immunosuppression. Classic signs and symptoms are hemoptysis, nasal mucosal ulceration, and saddle-nose deformity along with kidney damage (rapidly progressive glomerulonephritis, hematuria). Lung involvement that does not resolve with antibiotic therapy should raise a suspicious of a non-infectious etiology. Cytoplasmic antineutrophil cytoplasmic antibodies (c-ANCA) are associated with GPA and a specific subtype of these seen commonly in GPA are directed specifically against the anti-proteinase 3.The condition was originally named for Friedrich Wegener, who described the disease in 1936. As a response to Wegener's association with the German Nazi party, professional bodies and journals have replaced his name with a descriptive name. However, the older name is still often seen.

A 38-year-old woman comes to the emergency department because of shortness of breath and cough, productive of blood-tinged sputum for 3 days. She has been unresponsive to antibiotic therapy from her family doctor. Her temperature is 38.0°C (100.4°F), pulse is 92/min, respirations are 18/min, and blood pressure is 110/68 mm Hg. Laboratory studies show an elevated erythrocyte sedimentation rate (ESR) and circulating cytoplasmic anti-neutrophil cytoplasmic antibodies (c-ANCA). Urinalysis shows dysmorphic red cells and red blood cell casts. Which of the following antibodies is most likely to be found in the patient's serum? A. Anti-centromere antibodies B. Anti-myeloperoxidase antibodies C. Anti-proteinase-3 antibodies D. Anti-dsDNA antibodies E. Anti-mitochondrial antibodies

A. Henoch-Schönlein Purpura Major takeaway Henoch-Schönlein purpura is characterized by the classic clinical triad of palpable purpura petechia in dependent areas, migratory arthritis , and abdominal pain. Main explanation This child is suffering from a classic case of Henoch-Schönlein purpura (HSP). The classic clinical triad of HSP is palpable purpura/petechia (in dependent areas, see image below), migratory arthritis, and abdominal pain. However, the arthritis and abdominal pain can often precede the rash. Other symptoms include microscopic hematuria and proteinuria (which is only rarely in the nephrotic range). Intussusception and gastrointestinal hemorrhage can also occur. HSP is an IgA-mediated vasculitis with IgA and C3 immune complexes as the offending agents. It is often associated with a prior Streptococcal infection, but can follow an infection of various etiologies, including viruses and other bacteria.

A 4-year-old boy is brought to the emergency department by his mother because of a 1 day history of rash. She says that he had Streptococcal pharyngitis several weeks ago, but was in his normal state of health until 5 days ago when he began complaining of right hip pain. This subsided and was replaced by right knee and ankle pain one day ago. At the same time, he developed a rash of dark red spots scattered on his legs, buttocks, and forearms. He also began complaining of nausea and stomach pains 2 hours ago. Physical examination shows a purpuric rash located in dependent areas, and a swollen right knee without effusion, erythema, or warmth. Urinalysis shows microscopic hematuria with proteinuria. Serum electrolytes, including creatinine, are within normal limits. Which of the following is the most likely diagnosis? A. Henoch-Schönlein Purpura B. Rheumatic fever C. IgA nephropathy D. Reactive arthritis E. Systemic lupus erythematosus

D. Microscopic polyangiitis Major takeaway Microscopic polyangiitis (MPA) is one of several pulmonary/renal syndromes. Initial symptoms are usually very nonspecific and they include malaise, fever, and weight loss. The best initial test is p-ANCA, which is positive in 60% to 85% of patients. Treatment is glucocorticoids and cyclophosphamide. Main explanation Microscopic polyangiitis is a necrotizing vasculitis that typically affects capillaries, as well as small arterioles and venules. Initial symptoms are usually very nonspecific and they include malaise, fever, and weight loss. As the course of the disease progresses, vascular symptoms appear and dominate the clinical picture. Depending on the vascular bed involved, major clinical features include hemoptysis, hematuria, proteinuria, abdominal pain, muscle pain, and palpable cutaneous purpura. Microscopic polyangiitis (MPA) is one of several pulmonary/renal syndromes. There is considerable similarity with PAN and Wegener. All of them can give skin, renal, and neurological involvement. However, unlike PAN, MPA very frequently and severely involves the lung. Unlike Wegener, MPA does not involve the upper respiratory organs such as the nose and ear. MPA is a lung/kidney syndrome that is much more common than Goodpasture syndrome. The best initial test is p-ANCA, which is positive in 60%-85% of patients. The most accurate test is a biopsy. ESR and C-reactive protein are too nonspecific to be useful. Urinalysis will show proteinuria, hematuria, and red cell casts as it can in any form of glomerulonephritis. Treatment is glucocorticoids and cyclophosphamide.

A 55-year-old man comes to the emergency department because of dyspnea a productive cough with scarce mucoid sputum and blood. A year ago, he developed a fever, fatigue, and muscle pain. More recently, he experienced some hematuria and edema in his legs. He has a history of diabetes and hypertension. His temperature is 39.2°C (102.5°F), pulse is 118/min, respirations are 21/min, and blood pressure is 139/98 mm Hg. Bilateral diffuse wheezes are heard on auscultation. Chest radiographs shows diffuse infiltrates with an interstitial and alveolar pattern. Computerized tomography of the chest shows areas of bilateral ground glass appearance with micro-nodules. A biopsy of the kidney shows advanced sclerosis with formation of crescents. An anti-neutrophil cytoplasmic antibody (ANCA) investigation using indirect immunofluorescence was positive and disclosed a perinuclear pattern (p-ANCA). Which of the following is the most likely cause of the patient's symptoms? A. Churg-Strauss syndrome B. Goodpasture syndrome C. Granulomatosis with polyangiitis D. Microscopic polyangiitis E. Polyarteritis Nodosa

D. Microscopic polyangiitis Major takeaway Microscopic polyangiitis is a type of antineutrophil cytoplasmic autoantibody (ANCA)-associated vasculitis that mainly affects small-vessels. Biopsy of an artery shows necrotizing vasculitis without granulomas. Main explanation This patient presenting with constitutional symptoms, lung involvement, and nephritic syndrome, in association with a positive test for antineutrophil cytoplasmic autoantibody (ANCA) and biopsy showing vasculitis without granulomas, most likely has microscopic polyangiitis (MPA). It is a type of small-vessel necrotizing vasculitis characterized by fatigue, fever, weight loss, arthralgias, rhinosinusitis, cough, dyspnea, renal insufficiency, purpura, and neurologic dysfunction. MPA is a subtype of ANCA-associated vasculitides which include granulomatosis with polyangiitis (GPA), microscopic polyangiitis (MPA), and eosinophilic granulomatosis with polyangiitis (EGPA). Presentation of ANCA-associated vasculitides can be similar, and biopsy can assist in establishing the diagnosis. Biopsy showing necrotizing vasculitis without granulomas supports the diagnosis of MPA. Other distinguishing factors include nasopharyngeal involvement (e.g. nasal septum) seen mostly in GPA, and atopic history seen in eosinophilic granulomatosis with polyangiitis. ANCA-associated vasculitides are often termed pauci-immune due to the absence of immune complexes under immunofluorescence and electron microscopy.

A 40-year-old man comes to the clinic due to fatigue, cough and episodic blood in his urine during the past 6 months. He states he "hasn't been feeling like myself lately." Over the past 2 months, he has had a chronic cough which he attributed to an upper respiratory infection. His medical history is insignificant, and he does not smoke or use illicit drugs. On physical examination, wheezing is heard diffusely over the lung fields. Examination of the lower extremities reveals diffuse palpable purpura. Laboratory studies are obtained and shown below: Arterial biopsy of an affected region reveals focal transmural necrotizing lesions without granulomatous inflammation. Which of the following is the most likely diagnosis? A. Behçet syndrome B. Granulomatosis with polyangiitis C. Immunoglobulin A vasculitis D. Microscopic polyangiitis E. Eosinophilic granulomatosis with polyangiitis F. Acute postinfectious glomerulonephritis G. Giant cell arteritis

A. Antigen-antibody complexes Major takeaway Antigen- antibody complexes are thought to be the mediators of polyarteritis nodosa, a medium-vessel systemic vasculitis frequently associated with hepatitis B virus infection. Main explanation Polyarteritis nodosa (PAN) is a systemic vasculitis that affects medium-sized muscular arteries and is strongly associated with Hepatitis B (HBV) infection, although most cases are idiopathic. PAN is mediated by the formation of antigen-antibody complexes, and the mechanism of why this leads to medium-sized vessel inflammation is unknown. Inflammation can begin in the tunica intima and progress to include the entire wall, eventually destroying the elastic laminae and causing fibrinoid necrosis. Patients with PAN are typically characterized by systemic symptoms that include fatigue, weight loss, weakness, fever, and arthralgias. In addition, cutaneous features include tender erythematous nodules, purpura, ulcers, bulbous or vesicular eruptions, and livedo reticularis. Other systems that are affected include cardiac, renal, central and peripheral nervous systems, and the gastrointestinal system. PAN appears to spare the lungs, large and small sized vessels, as well as veins, and this is not clearly understood. Once PAN is suspected, diagnosis is confirmed by completing laboratory studies, vessel biopsy, and histological studies. Due to the multisystem effects of PAN, early diagnosis is crucial for treatment. Once diagnosed, PAN is treated with corticosteroids as well as antiviral agents for patients with HVB-related PAN.

A 42-year-old man comes to the emergency department because of a 2-day fever and blurred vision which prevented him from working. The patient has not travelled recently and has not been in contact with any sick individuals. However, for the past month, he has had joint pain, tingling and numbness in his hands and feet, and low appetite with weight loss. He is hypertensive and auscultation shows a pericardial friction rub. A livedo reticularis pattern rash on his arms and legs is observed as well as tender lesions on the dorsum of his feet. Laboratory studies show increased C-reactive protein, leukocytosis, and the presence of hepatitis B surface antigen. Which of the following immunological mechanisms is the most likely to cause the patient's symptoms? A. Antigen-antibody complexes B. Complement activation C. Cytotoxic T-cell mediated killing D. Immunoglobulin E production E. Overproduction of eosinophils

A. Pericardial effusion Major takeaway Pericardial effusions are associated with a variety of causes, including autoimmune disorders and infectious pericarditis. If pericardial effusion leads to cardiac tamponade, patients may have pulsus paradoxus, or a decrease in systolic blood pressure during inspiration of more than 10 mm Hg. Main explanation Certain ECG findings are characteristic for pericardial effusion . In particular, electrical alternans (shown by arrows) is highly specific for pericardial effusion (usually in association with cardiac tamponade) but not particularly sensitive. This pattern, characterized by beat-to-beat changes in the QRS axis in the limb and precordial leads, is caused by swinging of the heart within the accumulated pericardial fluid. Other common findings on ECG include sinus tachycardia and low QRS voltage. This patient's history is also highly suggestive of pericardial effusion, as effusions can be associated with autoimmune disease, viral or infectious pericarditis, and malignancy. She also has the finding of pulsus paradoxus (decrease in systolic blood pressure during respiration by more than 10 mm Hg), which occurs with cardiac tamponade. During inspiration, increased venous return to the right side of the heart causes bulging of the interventricular septum into the left ventricular cavity, leading to decreased left ventricular pressure and stroke volume. The left ventricle can normally undergo compensatory expansion; however, in the presence of a pericardial effusion causing tamponade, the left ventricle cannot compensate adequately during inspiration, leading to a more pronounced drop in blood pressure.

A 42-year-old woman comes to the emergency department because of chest pain, dyspnea, and lightheadedness. She recently recovered from a presumed viral upper respiratory infection and has a 10-year history of systemic lupus erythematosus. Physical examination shows a decrease in systolic blood pressure by 20 mm Hg during inspiration. An ECG is shown. Which of the following is the most likely diagnosis? A. Pericardial effusion B. Mitral stenosis C. Tricuspid regurgitation D. Aortic stenosis E. Aortic dissection

E. Tension pneumothorax Major takeaway Tension pneumothorax is characterized by decreased breath sounds and hyperresonance to percussion in the affected hemithorax. It also causes systemic hypotension with distended neck veins, and should be treated promptly. Main explanation Given the clinical setting, it is clear that this patient is most likely in cardiogenic shock . Decreased breath sounds, hyperresonance to percussion, systemic hypotension with distended neck veins are characteristic of tension pneumothorax. The stab injury most likely resulted in the introduction of air into the pleural space, leading to decreased breath sounds and hyperresonance. Expansion of this space resulted in a mediastinal shift away from the injury and a "kink" in the inferior vena cava, compromising blood flow to the heart. This resulted in distended neck veins and hypotension. Tension pneumothorax is a life-threatening emergency and should be treated with immediate insertion of a chest tube. Time is of the essence: if the clinical signs are obvious, do not wait to get a chest X-ray, and instead decompress the pneumothorax immediately.

A 43-year-old man comes to the emergency department because of a stab wound to the right side of his chest 20 minutes ago. He says that in addition to right-sided chest pain, he feels shortness of breath. His temperature is 36.8°C (98°F), pulse is 114/min, respirations are 26/min, and blood pressure is 88/62 mm Hg. He is having trouble finishing his sentences due to his shortness of breath. Pulmonary examination shows right-sided hyperresonance and pulmonary auscultation shows decreased breath sound also on the right-hand side. His neck veins are also distended. Which of the following is the most likely diagnosis? A. Cardiac tamponade B. Flail chest C. Hemothorax D. Pleural effusion E. Tension pneumothorax

C. Percutaneous coronary intervention Major takeaway Myocardial infarction requires urgent treatment for the best outcomes. Fibrinolytic drugs and percutaneous coronary intervention are the definitive steps in the management of STEMI. Significant hypertension (generally, >180/100 mm Hg) is a contraindication to the administration of fibrinolytic drugs. Main explanation This patient has multiple risk factors for myocardial infarction (MI): age, diabetes, hypertension, and male sex. Likewise, his signs and symptoms are consistent with this diagnosis. The first goal in the management of acute MI is to rapidly diagnose the condition and start therapy directed toward restoration of perfusion as soon as possible to salvage as much of the jeopardized myocardium as possible. This may be accomplished through medical (contraindicated in severely hypertensive patients) or mechanical means, such as percutaneous coronary intervention (PCI). During PCI, a cardiologist feeds a deflated balloon on a catheter from the inguinal femoral artery or radial artery up through blood vessels until they reach the site of the blockage in the coronary vasculature. At this point, contrast is injected to visualize the extent of stenosis. Thus, PCI is diagnostic as well as therapeutic. If this were a case of severe Prinzmetal angina, and no stenotic lesions were found, pharmaceutical challenge could be performed in this setting to diagnose the variant angina. Therapy consists in angioplasty: inflation of a balloon to open the artery and allow blood flow. Stents or scaffolds may be placed at the site of the blockage to hold the artery open. Tissue plasminogen activator is a fibrinolytic drug and heparin is an anticoagulant - neither is indicated in this patient because of his significant hypertension . Hypertension of this magnitude is a risk factor for hemorrhagic stroke . Additionally, percutaneous coronary intervention is the gold standard of treatment.

A 60-year-old man is brought to the emergency department because of crushing substernal chest pain for the past 45 minutes. He received 325 mg of aspirin en route. Nitroglycerin does not relieve his pain. He has a history of diabetes and hypertension. Medications include carvedilol and sildenafil. His temperature is 36.8°C (98°F), pulse is 99/min, respirations are 18/min, and blood pressure is 192/88 mm Hg. He appears diaphoretic. ECG shows ST-segment elevation in leads V1, V2, and V3. Which of the following is the most appropriate next step in management? A. IV heparin B. IV tissue plasminogen activator (tPA) C. Percutaneous coronary intervention D. Repeat ECG E. Troponin T level

D. Endothelial cells Major takeaway Atherosclerosis is a pathologic process that mainly affects medium to large arteries. The pathogenesis begins with an endothelial injury, followed by chronic inflammation and the formation of fibrous plaques that obstruct the lumen. Main explanation This patient's presentation and ECG findings are consistent with acute myocardial infarction , which given the patient's risk factors, is most likely due to atherosclerosis of the coronary arteries. Atherosclerosis is a pathologic process that affects arteries throughout the body including the aorta, coronary, cerebral, and peripheral arteries. Atherosclerosis is the hardening of any artery (but usually medium-large arteries) which is caused by the buildup of plaques (termed "atheromas"). Atherosclerosis can begin as early as childhood with the development of fatty streaks and advances with aging. The current hypothesis of the pathogenesis is the response-to-injury model, which is as follows: Endothelial cell injury, due to hypertension, smoking, hyperlipidemia, homocysteine or diabetes, is the first step in pathogenesis. Injury to endothelial cells results in endothelial dysfunction, increased membrane permeability, leukocyte adhesion and thrombosis. Lipoproteins, mainly oxidized LDL and cholesterol crystals, aggregate in the blood vessel wall. Platelets (choice C) adhere to exposed collagen and are activated. Monocytes (choice E) adhere to the endothelium, migrate into the intima, and differentiate into macrophages. Macrophages (choice A) engulf LDL, becoming foam cells (macrophages ingest LDL and become laden withlipids, which gives them a foamy appearance). The macrophages with engulfedlipids eventually undergo apoptosis (fatty streak) and release toxic oxidized LDL and inflammatory cytokines. At the same time, growth factors and inflammatory cytokines released from the activated platelets, macrophages, and endothelial cells then recruit smooth muscle cells (choice B). The growing smooth muscle secretes collagen, proteoglycans, and elastin fibrous cells, which all help form a wall around the fatty streak. The lipid core center surrounded by a fibrous cap is termed "fibrous plaque." When the formed fibrous plaques rupture due to shearing forces of blood flow, thrombogenic material (e.g., collagen) is exposed, which allows platelets to adhere to it. This buildup of platelets eventually occludes the artery and results in myocardial infarction, as seen in this patient.

A 60-year-old man is brought to the emergency department due to acute onset of chest pain. Medical history is significant for smoking 1 pack per day for the past 30 years, diabetes mellitus type 2, and hypertension. An ECG shows ST-segment elevation on V1-V4 leads. Cardiac catheterization is performed and shows 80% occlusion of the left anterior descending artery. The first step in the pathogenesis of this patient's condition most likely involves which of the following cell types? A. Platelets B. Monocytes C. Macrophages D. Endothelial cells E. Smooth muscle cells

A. Acute pericarditis Major takeaway Acute pericarditis is an inflammation of the pericardium which results in the cardinal signs of chest pain, pericardial friction rub on auscultation, and the changes of generalized ST-elevation and/or PR-depression on the ECG. Main explanation Acute pericarditis is inflammation of the pericardium, the sac surrounding the heart that consists of the serous and fibrous layers. This condition is characterized by chest pain, pericardial friction rub, and serial ECG changes. Other symptoms may include fever, dyspnea, tachypnea, cough, and dysphagia. Laboratory results typically show elevated ESR and CRP levels. The ECG changes can be organized into four stages; however, the significant changes of the ECG are the diffuse concave upward ST-segment elevation and/or the PR-segment depression. (Stage 1: Shows the onset of acute pain and is the most relevant in an emergency setting. ECG changes include diffuse ST-elevation and/or PR-wave depression; Stage 2: Shows an ST-segment return to baseline and flattened T-waves occuring days later; Stage 3: Shows inverted T-waves, no-Q wave formation; Stage 4: Shows return to baseline.)

A 44-year-old man comes to the emergency department because of sudden chest pain and difficulty breathing for the past three hours. The pain is felt in the retrosternal area and radiates up to his left shoulder and arm; it worsens on inspiration and is relieved when leaning forward. Physical examination shows no abnormalities. Pericardial friction rub is heard on auscultation. Laboratory results show elevated erythrocyte sedimentation rate (ESR) and C-reactive protein (CRP) levels. An ECG is taken and shown below. Which of the following is the most likely diagnosis? A. Acute pericarditis B. Angina pectoris C. Aortic stenosis D. Infective endocarditis E. Myocardial infarction

(B) PR depression Answer B (correct answer): Pericarditis is inflammation of the pericardial lining of the heart. Most cases are autoimmune with an unknown trigger, although other causes include viral infection, rheumatologic disorders (lupus, rheumatoid arthritis) and post myocardial infarction. Pericarditis is largely a benign condition but may mimic more serious disorders such as myocardial ischemia. The diagnosis is based on clinical symptoms and ECG findings - there are no diagnostic imaging or laboratory tests. The pain of pericarditis is usually present at rest. It worsens with lying flat and improves with sitting upright. The pain is often worse with inspiration (pleuritic). Cardiac exam may reveal a friction rub. ECG may show PR depressions and diffuse ST elevations involving multiple difference anatomic territories of the left ventricle. Answer A (incorrect answer): ST elevations limited to leads V1 to V4 are seen in anterior wall myocardial infarction. Answers C, D (incorrect answers): Pain that worsens with exercise and improves with rest is seen in myocardial ischemia from angina. Answers E (incorrect answer): Pulsus parodoxus is a fall in blood pressure with inspiration. This is seen in pericardial tamponade.

A 44-year-old man is evaluated for chest pain. Which of the following suggests a diagnosis of uncomplicated pericarditis? (A) ST elevations in leads V1 to V4 (B) PR depression (C) Pain worsened with exertion (D) Pain improved by rest (E) Pulsus paradoxus

C. Pulmonary embolism Major takeaway Risk factors for pulmonary embolism include obesity, malignancy, immobilization, and oral contraceptive use. Computed tomography scan may show a wedge-shaped pulmonary infarct. Main explanation Pulmonary embolism is a disease characterized by an embolus to the pulmonary vasculature commonly associated with deep venous thrombosis. Pulmonary embolism is characterized by acute onset shortness of breath, dyspnea, tachycardia, and tachypnea. In this clinical vignette, pulmonary embolism is the most likely diagnosis. This patient has several risk factors including obesity, oral contraceptive use, and recent immobility as suggested by a recent knee sprain. The classic finding on computed tomography scan is a wedge-shaped infarct as shown in this patient. A small embolism can reach the distal branches of the pulmonary artery and result in infarction of the pulmonary parenchyma supplied by those branches.

A 44-year-old woman comes to the emergency department because of sudden onset shortness of breath and right-sided chest pain with inspiration for the past 2 hours. She has a history of obesity and oral contraceptive use. She sprained her right knee a month ago and has been gradually recovering. Her temperature is 37°C (98.6°F), pulse is 110/min, respirations are 24/min, and blood pressure is 120/80 mm Hg. Physical examination shows a right leg which is swollen, tender, and erythematous. Lung sounds are clear and equal bilaterally. A computed tomography scan is done and shown below: Which of the following is the most likely diagnosis? A. Bacterial pneumonia B. Bronchiectasis C. Pulmonary embolism D. Sarcoidosis E. Tuberculosis

A. Longitudinal cleavage of the aortic media Major takeaway Aortic dissection is a potentially devastating vascular condition caused by longitudinal cleavage of the aortic tunica media created by a dissecting column of blood. The primary risk factor is uncontrolled hypertension and patients typically present with sudden onset sharp or tearing chest pain radiating to the back. Main explanation This patient in this vignette has concerning features for an aortic dissection given her risk factors of uncontrolled hypertension, in addition to the associated clinical features of a widened pulse pressure and characteristic neurologic deficits. Patients with aortic dissections classically present with sudden onset, sharp or tearing chest pain that radiates to the back, syncope, widened pulse pressure, and occasionally differing blood pressure in the upper extremities with associated neurologic deficits. Aortic dissections are caused by prolonged hydrodynamic shear stress on the intima of the vessel, which eventually creates a tear in the intimal layer and a column of blood gains access into the aortic media. With repetitive hydrodynamic stress the column of blood longitudinally tears through the media either retrograde (towards the heart), anterograde (away from the heart), or both. Eventually, the hematoma compresses on adjacent structures depending on where the dissection travels. For example, it may compress the brachiocephalic trunk or carotid arteries causing a stroke, or if proximal it may compress the right coronary artery causing an inferior wall MI, as in this patient with changes consistent with an NSTEMI. The hematoma may progress into the pericardium, causing pericardial tamponade. The most common risk factor aortic dissection is uncontrolled hypertension, likely precipitated by her chronic cocaine. Genetic risk factors include a history of connective tissue diseases likely Ehlers-Danlos syndrome or Marfan syndrome, which predispose to an already weakened aorta from thoracic aortic aneurysms. Infectious or inflammatory risk factors include Takayasu arteritis, giant cell arteritis, Behçet disease, and syphilis, which also weaken the aortic intima and predispose to intimal tearing.

A 45-year-old woman comes to the emergency department after experiencing several episodes of non-bilious and non-bloody emesis, followed by a sudden onset of intense chest pain radiating to the back. She also notes that she has left arm weakness. The patient states that prior to today, she has been in her usual state of health. She drinks a pint of vodka daily and regularly uses intranasal cocaine. She reports last using cocaine this morning. Her temperature is 37.0 °C (98.6 °F), pulse is 102/min, respirations are 22/min, blood pressure is 210/120 mmHg, and oxygen saturation is 96% on room air. The patient appears disheveled and in significant distress. Physical examination shows 1+ peripheral pulses in the right upper extremity and 2+ pulses in the left upper extremity. Neurological examination shows 3/5 power in the left upper and lower extremities compared to 5/5 power in the right upper and lower extremities. Laboratory markers demonstrate a troponin of 0.7 ng/ml and a WBC of 12,000/mm3. ECG demonstrates ST-depressions in leads II, III, and aVF. Which of the following is the most likely underlying mechanism of this patient's condition? A. Longitudinal cleavage of the aortic media B. Full thickness perforation of the esophagus C. Defect in lung ventilation/perfusion ratio D. Atherosclerotic rupture of a right coronary artery plaque E. Aneurysmal rupture of the left middle cerebral artery

A. Anterior wall myocardial infarction Major takeaway Wellens' syndrome is an ECG manifestation that is specific for severe left anterior descending artery stenosis in patients with unstable angina. It is characterized by symmetric, deeply inverted T-waves in V2 and V3. Main explanation This patient has Wellens' syndrome, which is an ECG manifestation that is specific for severe left anterior descending artery stenosis in patients with unstable angina. It is characterized by symmetric, deeply inverted T-waves in V2 and V3. A less common variation shows biphasic T-waves in V2 and V3. Additional diagnostic criteria include a history of chest pain with normal to minimally elevated cardiac enzymes, no Q-waves, no ST-elevation, and no loss of precordial R-waves. Research suggests that over 50% of patients with Wellens' syndrome have a complete or near-complete occlusion of the left anterior descending coronary artery and that approximately three-quarters will develop an anterior wall myocardial infarction. Consequently, further evaluation in the form of coronary angiography followed by immediate intervention and revascularization would be likely next steps in management.

A 47-year-old man comes to the emergency department because of a 40-minute episode of central chest pain that occurred 2 hours ago and has since subsided. This is the first time that the patient has experienced this type of pain. His troponin and CK-MB levels are normal and an electrocardiogram (ECG) is shown below. Which of the following types of acute myocardial infarction is the patient at most risk for? A. Anterior wall myocardial infarction B. Inferior wall myocardial infarction C. Lateral wall myocardial infarction D. Posterior wall myocardial infarction E. Septal myocardial infarction

D. Small cell carcinoma Major takeaway Small cell carcinoma tends to occur centrally in larger airways, with neuroendocrine differentiation and cells resembling lymphocytes (small, little cytoplasm, large nuclei, even chromatin). Main explanation Small cell carcinoma tends to occur centrally in larger airways, with neuroendocrine differentiation and cells resembling lymphocytes (small, little cytoplasm, large nuclei, even chromatin). Secretion of adrenocorticotrophic hormone (ACTH) can cause increased cortisol levels, leading to Cushing syndrome. Small cell carcinoma is generally highly metastatic and has a very poor prognosis. For this reason, it is most commonly treated with chemotherapy rather than surgery.Other paraneoplastic syndromes associated with small cell lung cancer are the syndrome of inappropriate anti-diuretic hormone secretion, Lambert-Eaton syndrome, carcinoid syndrome, and superior vena cava (SVC) syndrome due to SVC obstruction. Common symptoms of small cell lung cancer include cough, dyspnea, weight loss, and debility. >70% of patients with small-cell carcinoma is characterized by metastatic disease, common sites include liver, adrenals, bone, and brain

A 48-year old man comes to the office because of rapid weight gain for 7 months. He states that he has put on 9-kg (20-lb) in weight despite increasing his exercise routine and dieting. He says he has also noticed that he has been sweating more than usual and that he has had a constant cough for the past 4 months. Medical history includes smoking 2 packets of cigarettes per day for 30 years. A chest X-ray is obtained. Subsequent biopsy and histology show lymphocyte-like cells differentiating in a neuroendocrine pattern. Which of the following is the most likely diagnosis? A. Adenocarcinoma B. Large cell carcinoma C. Pulmonary carcinoid D. Small cell carcinoma E. Squamous cell carcinoma

D. Polyarteritis nodosa Major takeaway Polyarteritis nodosa is a systemic medium-vessel vasculitis affecting middle aged men that is often associated with hepatitis B infection. Main explanation This patient presenting with constitutional symptoms, renal involvement, mononeuritis multiplex, and positive serology for hepatitis B infection most likely has polyarteritis nodosa (PAN). PAN is medium-vessel vasculitis typically seen in middle-aged men that is often associated with hepatitis B/C and hairy cell leukemia. Biopsy of the affected arteries will show transmural inflammation of the arterial wall with fibrinoid necrosis. The absence of perinuclear antineutrophil cytoplasmic antibodies (p-ANCA), no evidence of pulmonary involvement, and the positive serology for hepatitis B helps to narrow the diagnosis.

A 50-year-old man comes to the clinic due to fever, weight loss and weakness in his right foot and left hand for the past 2 months. During this period, he lost 4.5-kg (10-lb) without intention or changes in his diet. He has no jaw pain, change in vision, or headaches. He last visited his primary care physician more than 10 years ago, and his medical history is significant for IV drug use in his 20s. He no longer uses illicit drugs and does not smoke cigarettes. Temperature is 38.0°C (100.4° F), pulse is 80/min, and blood pressure is 145/90 mm Hg. Physical examination shows high-stepping gait when the patient lifts the right foot and decreased strength in the extensors of the left hand. There is no pain with palpation of the scalp. Cardiopulmonary exam shows no abnormalities. Skin examination shows purpura. Laboratory studies show the following: Which of the following is the most likely diagnosis? A. Microscopic polyangiitis B. Granulomatosis with polyangiitis C. Temporal arteritis D. Polyarteritis nodosa E. Thromboangiitis obliterans F. Eosinophilic granulomatosis with polyangiitis

D. Coxsackie B Major takeaway Myocarditis is inflammation of the muscle of the heart, and is most often caused by viruses such as coxsackie B, adenovirus, and parvovirus B19. Main explanation Coxsackie B virus is typically characterized by fever, sore throat, and headache. Most coxsackievirus infections last for 2-10 days. Cardiac symptoms typically occur two weeks later, and since not everyone who gets coxsackie B virus gets myocarditis, it is thought to develop both because of direct tissue injury by the virus. Parts of the virus also resemble the cardiac protein myosin, so a patient's immune system ends up attacking the heart as well as the virus.This patient has signs and symptoms concerning for myocarditis, inflammation of the muscle of the heart. Myocarditis has a wide range of presentations, from subclinical disease to sudden cardiac death. Common symptoms include chest pain and generalized fatigue, other symptoms typically depend on the etiology.

A 50-year-old man comes to the emergency department because his heart "is beating really quickly" and diffuse chest pain. He has never had symptoms like this before and has been generally healthy, except for a recent febrile illness two weeks ago during which he had a fever, headache, and sore throat for two days. His sick contacts include his two children, who had similar symptoms before he developed a fever. Cardiac examination shows non-reproducible chest pain with no murmurs, rubs, or gallops on auscultation. The rest of the physical examination is significant only for an ill-appearing man with bilateral lower extremity edema. Which of the following is the most likely cause of his presentation? A. Escherichia coli B. Staphylococcus epidermidis C. Trypanosoma cruzi D. Coxsackie B E. Echovirus

E. Supportive therapy only Major takeaway Myocarditis is inflammation of the heart muscle, most often caused by a viral infection. As such, therapy is often supportive rather than specific to the etiology. Main explanation This patient currently requires supportive therapy only. Myocarditis is most often caused by viral etiology, so treatment is often non-specific and supportive. If the disease has progressed to heart failure, a typical presenting sign, the corresponding therapy is indicated. At the moment, the patient only has chest pain and a history concerning for myocarditis.Coxsackie virus typically presents as a fever, sore throat, and headache. As the fever breaks, patients may develop a rash. Most coxsackie virus infections last for 2-10 days. Cardiac symptoms typically occur two weeks later. Not everyone who gets coxsackie B virus gets myocarditis -- it is thought to develop both because of direct tissue injury by the virus and also because of an autoimmune reaction.

A 50-year-old man comes to the emergency department because his heart "is beating really quickly" and he has diffuse chest pain. He has never had symptoms like this before and has been generally healthy, except for a recent febrile illness about two weeks ago during which he had a fever, headache, and sore throat for two days. His sick contacts include his two children, who had similar symptoms before he developed a fever. He denies any recent joint pain or skin rashes. Cardiac examination shows non-reproducible chest pain with no murmurs, rubs, or gallops on auscultation. Which of the following is a reasonable treatment choice for his condition? A. Antibiotic therapy B. Immunosuppressive therapy C. Rate control D. Rhythm control E. Supportive therapy only

(A) Constrictive pericarditis Answer A (correct answer): This patient has clinical features consistent with constrictive pericarditis from prior chest radiation. This disorder occurs when the pericardium becomes scarred and loses elasticity. This leads to impaired ventricular filling. Although a rare disorder, it is most often seen following acute pericarditis, cardiac surgery or chest radiation. Kussmaul sign—a rise in jugular venous pressure with inspiration—is a classic finding. Cardiac exam may show a pericardial "knock" in early diastole. Chest imaging may show calcifications of the pericardium. Left and right ventricular function is normal on echocardiography as ventricular contraction is not impaired. Clinical pearl: Restrictive cardiomyopathy and constrictive pericarditis often have prominent right heart failure symptoms as seen in this question. Jugular venous pressure can be markedly elevated. The abdomen and liver may be distended mimicking liver disease. A "pulsatile liver" is a classic finding in severe right heart failure as tricuspid regurgitation is transmitted to the abdomen. Answer B (incorrect answer): In ischemic cardiomyopathy, the left ventricular ejection fraction is reduced. Answer C (incorrect answer): Massive pulmonary embolism can lead to right heart failure. The presentation is usually acute (i.e. not evolving over one month as in the question). Chest pain is usually present. ECG often demonstrates sinus tachycardia or, less commonly, a right heart strain pattern (incomplete right-bundle-branch block and S1Q3T3). If right heart failure signs and symptoms occur, the right ventricle will be dilated and hypokinetic on echocardiography. Answer D (incorrect answer): Portal hypertension occurs in liver cirrhosis and may cause abdominal distension, ascites, and lower extremity edema. Elevated jugular venous pressure should not be present and this feature is key to distinguishing portal hypertension from right heart failure. In clinical practice, patients with rare causes of right heart failure (constrictive pericarditis, restrictive cardiomyopathy) are sometimes misdiagnosed with a primary liver disorder. This is especially true in patients who have risk factors (viral hepatitis, alcohol consumption) for chronic liver disease. The liver can become cirrhotic from long-standing right heart failure. Answer E (incorrect answer): Mitral regurgitation causes a holosystolic murmur at the cardiac apex. The murmur decreases with inspiration.

A 52-year-old man presents with abdominal discomfort. He has a history of Hodgkin's lymphoma treated with radiation and chemotherapy 10 years ago. He drinks alcohol socially on weekends. His only travel outside the U.S. was a trip to Costa Rica 4 years ago. He reports fatigue and exertional dyspnea for the past one month. His abdomen always has a "full" feeling and his pants are tight. Blood pressure is 124/72 mmHg and pulse is 84/min. Lungs are clear. Jugular venous pressure is 14 mmHg (normal <8) and rises during inspiration. On cardiac exam, there is an early diastolic heart sound. The abdomen is distended and the liver is pulsatile. 2+ pitting edema is present. Echocardiography shows normal left and right ventricular function. Which of the following is the most likely diagnosis? (A) Constrictive pericarditis (B) Ischemic cardiomyopathy (C) Pulmonary embolism (D) Portal hypertension (E) Mitral regurgitation

C. Duodenal ulcer Major takeaway Patients with duodenal ulcers classically present with gnawing upper abdominal pain that is most severe several hours post-prandially, and is temporarily relieved by eating. Main explanation A duodenal ulcer is a mucosal breach in the first part of the small intestine. Up to 90% of duodenal ulcers and 60% of gastric ulcers are a result of chronic inflammation due to Helicobacter pylori bacterial infection. NSAIDs like aspirin and ibuprofen, and glucocorticoids can cause or worsen peptic ulcers due to COX-1 blockade and prostaglandin downregulation. For duodenal ulcers, the classic upper abdominal pain occurs when acid is secreted in the absence of a buffer. Since food-stimulated acid production occurs for three to five hours after a meal, but food is only present for two to three hours after a meal, the pain of a duodenal ulcer occurs in the window two to five hours after meals, or when the stomach is empty. Pain symptoms are also common between 11pm and 2am, when the body's circadian production of acid is at its greatest. The pain may be described as burning, hunger-like, or gnawing, but it can also be vague. Patients with duodenal ulcers may also be asymptomatic until a complication, such as hemorrhage or perforation, occur. Any patient presenting with a suspected peptic ulcer should be assessed for red flags that suggest underlying malignancy requiring urgent endoscopy

A 52-year-old woman comes to the clinic because of regular episodes of epigastric abdominal pain. The pain occurs a few hours after eating, and is severe (8/10) when it occurs. She has found that eating sometimes helps relieve the pain. Past medical history is significant for obesity and smoking. Physical examination indicates only vague epigastric tenderness to palpation. Which of the following is the most likely diagnosis? A. Acute cholecystitis B. Choledocholithiasis C. Duodenal ulcer D. Gastric ulcer E. Pancreatitis

B. Adenovirus Major takeaway Myocarditis in the United States is most commonly caused by a viral infection, typically adenovirus, coxsackie B, parvovirus B19, or others. It often presents with systemic symptoms such as fatigue and chest pain, but may cause sudden death. Main explanation This patient has acute myocarditis , likely due to an adenovirus, or another virus that can result in a flu-like illness. Worldwide, the most common cause of myocarditis is Trypanosoma cruzi, a protozoan endemic to Central and South America. In the U.S., however, the most common cause is a viral infection. Common pathogens include adenovirus, parvovirus B19, coxsackie virus, HIV, enterovirus, rubella virus, polio virus, CMV and HHV-6. As most viral infections cannot be treated with directed therapy, symptomatic treatment is the only form of therapy for these forms of myocarditis. In the acute phase, supportive therapy, including bed rest, is indicated. For symptomatic patients, digoxin and diuretics may provide clinical improvement. For patients with moderate to severe disease, cardiac function can be supported by the use of ionotropes such as milrinone in the acute phase, followed by oral therapy with ACE inhibitors when tolerated. People who do not respond to conventional therapy are candidates for bridge therapy with left ventricular assist devices (LVADs). Heart transplantation is reserved for patients who fail to improve with conventional therapy.

A 54-year-old man comes to the emergency department following a four-day history of left-sided chest pain and shortness of breath. One week ago, he experienced upper respiratory symptoms along with myalgias and general fatigue. He has no known past medical history. He has not traveled outside the US. His temperature is 38.1°C (100.6°F), pulse is 104/min, respirations are 17/min, oxygen saturation is 94% on room air, and blood pressure is 100/72 mm Hg. Physical examination shows an ill-appearing man with bibasilar rales, jugular venous distention of 11 cm with no murmurs, rubs, or gallops, and 1+ bilateral pitting edema of the lower extremities. His labs are within normal limits and his blood cultures are negative. An echocardiogram shows an ejection fraction of 35%. Which of the following is the most likely cause of this patient's condition? A. Trypanosoma cruzi B. Adenovirus C. Coxsackie A virus D. Staphylococcus aureus E. Corynebacterium diphtheriae

D. Ischemia caused by blockage of the left inferior division of the middle cerebral artery Major takeaway Ischemic stroke is commonly caused by embolic disease due to atrial fibrillation. Another significant cause of an ischemic stroke is atherosclerosis. Right-sided motor and sensory deficits are caused by contralateral left-sided cerebral involvement which can include Wernicke and Broca area. Main explanation This patient is experiencing a classic presentation of ischemic stroke involving the inferior division of the left middle cerebral artery. He has a sudden onset of right-sided hemiparesis and sensory deficits as well as Wernicke aphasia. Wernicke aphasia is a receptive type of aphasia that results in the inability to read, write, and repeat sentences with poor comprehension; fluency is intact, however. Patients are unaware of their language errors as the speech is preserved but the language content is not correct. The incorrect language content can range from insertions of non-existent or wrong words to a profuse jargon usage.Other symptoms of a stroke include the inferior division of the middle cerebral artery (MCA), which includes contralateral homonymous hemianopia and apraxia. The patient's history of atrial fibrillation should raise the suspicion of an ischemic stroke brought on by an embolus originating from the heart. Patients may also have a history of diabetes and hypertension, which are also considered significant risk factors for stroke.Diagnosis of stroke initially involves non-contrast CT imaging. If imaging shows no hemorrhage, consider aspirin. Patients already on an aspirin regimen may be given clopidogrel, ticlopidine, or dipyridamole.

A 55-year-old African American man comes to the emergency department because of sudden weakness and numbness on the entire right half of his body. Medical history includes atrial fibrillation and type 2 diabetes mellitus. Examination shows diminished motor strength and sensory deficit in his right extremities. He is unable to follow verbal or written commands and repeat phrases. His speech, however, is spontaneous and of good quality. Which of the following is the most likely cause of the patient's speech deficits? Elimination tool A. Intra-cerebral hemorrhage of the middle cerebral artery due to poor sugar control B. Intra-parenchymal hemorrhage due to rupture of the lenticulostriate vessels C. Intracranial hemorrhage into the subarachnoid space from disrupted vessel wall integrity D. Ischemia caused by blockage of the left inferior division of the middle cerebral artery E. Ischemia caused by blockage of the left superior division of the middle cerebral artery

E. Karyopyknosis, hypereosinophilia of myocytes, contraction band necrosis Major takeaway Following myocardial infarction , histopathological changes follow a specific and predictable pattern that can be used as a timeline. Troponin levels peak 18-24 hours post infarction, during which time there is ongoing coagulation necrosis, karyopyknosis, hypereosinophilic myocytes, and contraction band necrosis. Main explanation This patient had a massive coronary infarction. This question requires you to know the histological findings of the myocardium during various timeframes following initial infarction. His troponin levels are relatively high. Troponin levels peak between 18-24 hours post infarction (MI), and the severely elevated troponin levels seen with this patient point toward that time frame. Atypical chest pain or lack of chest pain makes timing the onset of myocardiaI onset difficult using symptoms alone, since patients' subjective impression of when it began may not be accurate. The timeframe 18-24 hours after MI has the following distinct histopathological features: ongoing coagulation necrosis, karyopyknosis (condensation of chromatin in the nucleus of a cell undergoing necrosis or apoptosis), hypereosinophilia of myocytes, contraction band necrosis in margins, and beginning of neutrophil infiltration. The table in the following link summarizes the timeframes and changes seen. https://en.wikipedia.org/wiki/Timeline_of_myocardial_infarction_pathology

A 55-year-old man comes to the emergency department because of chest pain that began about 18 hours ago. He describes it as substernal pressure with electrical shooting pains down his left arm. He has a past medical history of type II diabetes mellitus, hypertension, and hyperlipidemia. His medications include metformin, pravastatin, lisinopril and hydrochlorothiazide. Laboratory studies show a troponin concentration of 1.5 ng/mL. Despite comprehensive resuscitation efforts, the patient dies. An autopsy is performed and the following illustration represents the macroscopic appearance of the patient's heart.Which of the following best characterizes the histological findings of the patient's myocardium? A. Disintegration of dead muscle fibers, necrosis of neutrophils B. Glycogen depletion and waviness of fibers at border C. Granulation tissue formation D. Increased collagen deposition and decreased cellularity E. Karyopyknosis, hypereosinophilia of myocytes, contraction band necrosis

D. Ventricular tachycardia Major takeaway Myocardial infarction increases the likelihood of a number of conditions. The leading cause of acute mortality following myocardial infarction is ventricular tachycardia degenerating into ventricular fibrillation, pulseless electrical activity, and asystole. Main explanation The electrocardiogram in the question stem shows ST-elevations in leads V1-V6 and reciprocal changes (ST-depressions) in lead III and aVF. This is consistent with an anterior ST-elevation myocardial infarction (STEMI). In the hours and days following a MI, people are at increased risk for various arrhythmias including ventricular tachycardia and atrial fibrillation. They are also at risk for heart failure, heart block, and myocardial aneurysm or rupture. The leading cause of acute mortality is ventricular tachycardia degenerating into ventricular fibrillation, pulseless electrical activity, and asystole. Scar tissue formed following myocardial ischemia is thought to be arrhythmogenic. While this tissue is a risk factor for myocardial aneurysm and even myocardial rupture, ventricular tachycardia is the leading cause of death.

A 55-year-old man comes to the emergency department because of severe central chest pain for 2 hours. He says the pain is heavy in nature and radiates to his jaw. Medical history includes uncontrolled hypertension and type II diabetes mellitus. His temperature is 36.8°C (98°F), pulse is 97/min, respirations are 18/min, and blood pressure is 163/91 mm Hg. Cardiovascular examination is noncontributory. An ECG is obtained and it shows ST-elevations in leads V1-V6, as well as reciprocal changes in leads III and aVF. Which of the following complications is most likely to occur, and result in death during this admission? A. Atrial fibrillation B. Atrial flutter C. Heart failure D. Ventricular tachycardia E. Myocardial wall rupture

E. Skin infection, Varicella Major takeaway Chickenpox causes a pruritic vesicular rash with lesions in multiple stages in development. The most common complication in children is bacterial skin and soft tissue infection. Main explanation This boy has chickenpox , or primary varicella-zoster virus (VZV) infection. It is a highly contagious viral disease. Prodromal symptoms include fever and malaise, followed by development of the characteristic vesicular rash. Typically, lesions begin on the trunk and spread outwards to the extremities. On examination, they will be in different stages of development. The rash is generally extremely pruritic, and supportive measures should be taken for symptomatic relief. Oral antihistamines can be used to reduce itching, and fingernails should be cut closely to prevent excoriations. The most likely complication of chickenpox in children is bacterial skin and soft tissue infection, including cellulitis, myositis, and necrotizing fasciitis

A 56-year-old manis brought to the office because of a 3-day history of fever, malaise, and rash. The rash started on the head and chest and has since spread to the extremities. It is painful. A photograph of the lesions on his trunk is shown, with similar vesicular lesions present on his face and extremities. This patient is at greatest risk for which of the following conditions? A. Pneumonia B. Hepatitis C. Meningitis, Influenza D. Encephalitis, Covid 19 E. Skin infection, Varicella F. Meningitis, Parvo B19 G. Skin infection, Varicella

B. Chronic pancreatitis Major takeaway A diagnosis of chronic pancreatitis is suggested by upper abdominal pain, weight loss, and malabsorption in a patient with risk factors for acute pancreatitis. Patients with chronic pancreatitis may develop pancreatic exocrine insufficiency, characterized by fatty diarrhea (steatorrhea) and fat-soluble vitamin deficiency. Main explanation This patient most likely has pancreatic exocrine dysfunction secondary to chronic pancreatitis from chronic heavy alcohol consumption. Continuous inflammation or repeated episodes of acute pancreatitis results in replacement of normal pancreatic parenchyma with fibrotic tissue, leading to the loss of pancreatic exocrine acinar cells. These cells contain digestive enzymes (including trypsinogen) that are secreted into the pancreatic ducts and subsequently activated in the duodenum to digest food into its absorbable components. When these cells and enzymes are lost, dietarylipids cannot be absorbed. As a consequence, patients may experience features of fat malabsorption including fatty diarrhea (steatorrhea) and fat-soluble vitamin (A, D, E, & K) deficiency. Malabsorption of the fat-soluble vitamins may cause their respective deficiency symptoms:

A 58-year-old woman with a history of chronic alcohol use disorder comes to the office because of diarrhea, generalized weakness, and a 6.8-kg (15-lb) weight loss over the past 6 months. She reports intermittent dull upper abdominal pain that will last for days at a time and is not improved with antacids. After meals, she feels that her abdomen is distended. She characterizes her multiple daily bowel movements as greasy, foul-smelling, and oily. She recently was the driver involved in a minor traffic accident, which she attributes to worsening eyesight at night. Her temperature is 37.0°C (98.6°F), pulse is 78/min, respirations are 16/min, and blood pressure is 135/85 mmHg. Abdominal examination shows resonance to percussion throughout and a mildly tender epigastrium. Bowel sounds are hyperactive. Laboratory values show the following: Which of the following is the most likely etiology of this patient's diarrhea? A. Carcinoid tumor B. Chronic pancreatitis C. Celiac disease D. Chronic gastritis E. Pancreatic adenocarcinoma

A. Gastroesophageal reflux disease Major takeaway Gastroesophageal reflux disease (GERD) is a common cause of vomiting in very young children that almost always resolves on its own by 1 year of age. Main explanation Gastroesophageal reflux (GERD) is commonly seen in pediatric patients. GERD is the passage of gastric contents into the esophagus and is a normal physiologic process that occurs in healthy infants and children. Typically, it is first seen in the first few months of life and will resolve by 1-2 years of age. Most episodes are brief and do not cause symptoms or other complications such as esophagitis or poor weight gain. Typically, children with GERD are treated with small feeds, thickened formula, and avoiding overfeeding. If the patient does not improve by 18-24 months of age then the child should be re-evaluated. Only a small number of children require proton pump inhibitors.

A 6-month-old boy comes to the office because of spitting up his meals for the past five months. He is also often unsettled, with colicky behavior after feeding. Other than this, his mother has no concerns. He is developing normally, and his growth has been stabilized at the 90th percentile for his height and weight. Examination shows a well-appearing child. Abdominal examination shows no abnormalities. Which of the following is the most likely diagnosis? A. Gastroesophageal reflux disease B. Hypothyroidism C. Meckel's diverticulum D. Partial duodenal atresia E. Pyloric stenosis

E. Blockage of the left central retinal artery by cholesterol embolus Major takeaway Ocular ischemic syndrome is the constellation of ocular signs and symptoms secondary to severe, chronic arterial hypoperfusion to the eye. Main explanation This man is presenting to emergency department with what is likely a cholesterol embolus occlusion of his left central retinal artery. This diagnosis is suggested in the history: a sudden onset, persisting, painless loss of vision in an older man who has existing hypercholesterolemia and hypertension. Older age, male gender, hypercholesterolemia, and hypertension are four major risk factors for atherosclerotic disease and subsequent cholesterol emboli, especially during increased work such as climbing stairs. Furthermore, on fundoscopy, the man has a well-perfused macula ("cherry-red spot"), which is served by the choroid vessels, while surrounding retina, served by the central retinal artery, is not well perfused.

A 60-year old man comes to emergency department because of acute painless loss of vision in his left eye that came on early that morning, as he was going up a long flight of stairs at home. He says that he cannot see at all with his left eye, but his right eye is unaffected. He is currently taking atorvastatin and enalapril. A fundoscopic examination is performed and it shows that his left pupil dilates slightly when light is passed from his right to left eye. His macula is well-perfused, indicated by the presence of a bright red spot at its center, but there seems to be a bit of hazy translucency of the retina otherwise. Which of the following is your suspected mechanism of injury? A. Glycosylation of retinal blood vessels B. Closed angle glaucoma C. Congenital arteriovenous malformations D. Marcus-Gunn pupil resulting from a fall down the stairs E. Blockage of the left central retinal artery by cholesterol embolus

Major takeaway Gastroesophageal reflux disease (GERD) is the most common non-cardiac cause of chest pain. It is characterized by chest pain that may occur in a supine position, after meals, and along with a sour, acidic taste in the mouth. Main explanation Gastroesophageal reflux is the most common cause of non-cardiac chest pain and the most likely cause of this patient's chest pain. Typical symptoms include burning substernal pain associated with food, as well as increased discomfort in the supine position, nausea and vomiting, and a sour taste in the mouth due to acid regurgitation ("water brash"). Less common causes of non-cardiac, gastrointestinal related chest pain include esophageal motility disorders, esophageal rupture, or tears and esophagitis. Treatment is typically lifestyle changes and medications such as proton pump inhibitors, H2 receptor blockers or antacids with or without alginic acid. Surgery may be an option in those who do not improve. In the Western world between 10 and 20% of the population is affected.

A 60-year-old man comes to the emergency department because of sub-sternal chest pain for an hour. He has experienced this pain before, typically after heavy meals, during times of stress, and when he goes running in the park. He also feels the pain at night when he is lying in bed and has previously been woken from sleep by discomfort. He has not noticed any dyspnea, diaphoresis, or palpitations but is currently experiencing some nausea and a sour taste in his mouth. Medical history is non-contributory. ECG shows normal sinus rhythm. Which of the following is the most likely diagnosis? A. Achalasia B. Diffuse esophageal spasm C. Esophageal stricture D. Gastroesophageal reflux E. Stable angina

B. Small cell carcinoma Major takeaway Small cell carcinoma of the lung occurs almost exclusively in smokers and is associated with Lambert-Eaton myasthenic syndrome, due to the production of autoantibodies that crossreact with antigens at the neuromuscular junction. Main explanation A pulmonary nodule in a patient with a significant smoking history requires evaluation for malignancies of the lung. This patient's history of muscle weakness that improves with activity is consistent with Lambert-Eaton myasthenic syndrome (LEMS), a disorder that results from the production of autoantibodies that crossreact with calcium channels at presynaptic terminals of the neuromuscular junction. In conjunction with the finding of a nodule near the hilum, the presence of LEMS is suggestive of small cell carcinoma of the lung. These tumors are characterized by small cells with little cytoplasm that stain positive for keratin and epithelial markers as well as synaptophysin, chromogranin, and other markers of neuroendocrine differentiation. They are associated with a variety of paraneoplastic syndromes, including ectopic production of ACTH and ADH (vasopressin).

A 60-year-old man comes to the office because of a 6-month history of cough, dyspnea, and muscle weakness. The weakness is worst when he is resting and improves after he has walked around. He has smoked one pack of cigarettes daily for 40 years. An x-ray of the chest shows flattening of the diaphragm and elongation of the mediastinum as well as a 2-cm nodule near the hilum. Which of the following is the most likely diagnosis? A. Adenocarcinoma B. Small cell carcinoma C. Squamous cell carcinoma D. Large cell carcinoma E. Sarcoidosis

D. Fibrinolytic drugs Major takeaway Myocardial infarction requires urgent treatment for the best outcomes. Fibrinolytic drugs and percutaneous coronary intervention are the definitive steps in the management of STEMI. Significant hypertension (generally, >180/100 mm Hg) is a contraindication to the administration of fibrinolytic drugs. Main explanation This patient has multiple risk factors for myocardial infarction (MI): age, diabetes, hypertension, and male sex. Likewise, his signs and symptoms are consistent with this diagnosis. The first goal in the management of acute MI is to rapidly diagnose the condition and start therapy directed toward restoration of perfusion as soon as possible to salvage as much of the jeopardized myocardium as possible. This may be accomplished through medical (contraindicated in severely hypertensive patients) or mechanical means, such as percutaneous coronary intervention (PCI). During PCI, a cardiologist feeds a deflated balloon on a catheter from the inguinal femoral artery or radial artery up through blood vessels until they reach the site of the blockage in the coronary vasculature. At this point, contrast is injected to visualize the extent of stenosis. Thus, PCI is diagnostic as well as therapeutic. If this were a case of severe Prinzmetal angina, and no stenotic lesions were found, pharmaceutical challenge could be performed in this setting to diagnose the variant angina. Therapy consists in angioplasty: inflation of a balloon to open the artery and allow blood flow. Stents or scaffolds may be placed at the site of the blockage to hold the artery open.

A 60-year-old man is brought to the emergency department because of crushing substernal chest pain for the past 45 minutes. He received 325 mg of aspirin en route. Nitroglycerin does not relieve his pain. He has a history of diabetes and hypertension. Medications include carvedilol and sildenafil. His temperature is 36.8°C (98°F), pulse is 99/min, respirations are 18/min, and blood pressure is 125/80 mm Hg. He appears diaphoretic. ECG shows ST-segment elevation in leads V1, V2, and V3. Which of the following is the most appropriate next step in management? A. IV heparin B. Repeat ECG C. Percutaneous coronary intervention D. Fibrinolytic drugs E. Troponin T level

C. Myocardial wall rupture Major takeaway Myocardial infarction may be complicated by myocardial wall rupture 3-14 days after the initial infarction. This causes acute cardiac tamponade, which is characterized by Beck's triad: hypotension, jugular venous distension, and distant heart sounds. Main explanation This patient has developed cardiac tamponade (acute build-up of fluid in the pericardial space) five days after a myocardial infarction. This patient has all three features of the Beck triad of acute cardiac tamponade: hypotension, jugular venous distension, and distant heart sounds. Myocardial wall rupture occurs in about 3% of people following myocardial infarction, and most commonly occurs 3-14 days after the infarction. Pulsus paradoxus is an abnormally large decrease in systolic blood pressure and pulse wave amplitude during inspiration, and is a sign of acute cardiac tamponade. In this situation, urgent pericardiocentesis is required to drain the fluid around the heart and restore function. Histology shows extensive phagocytosis of dead cells, along with collagen deposition. Pericarditis is common in the first 2 days following a transmural myocardial infarction . The characteristic symptoms of pericarditis are pain, worse when lying flat, a pericardial rub, and a pericardial effusion (as shown on the echocardiogram).

A 60-year-old man is reviewed on the cardiology ward because of hypotension and prominent jugular vein distention for 10 minutes. He is on the ward because of a large myocardial infarction five days ago. Medical history includes type 2 diabetes mellitus, uncontrolled hypertension, and hypercholesterolemia. His temperature is 36.8°C (98°F), pulse is 112/min, respirations are 21/min, and blood pressure is 94/78 mm Hg. His pulse is weaker to palpation during inspiration, and cardiac auscultation shows muffled heart sounds. Which of the following is the most likely diagnosis? A. Arrhythmia B. Myocardial infarction C. Myocardial wall rupture D. Pericarditis E. Ruptured papillary muscle

D. Pericarditis Pericarditis is common in the first 2 days following a transmural myocardial infarction . The characteristic symptoms of pericarditis are pain, worse when lying flat, a pericardial rub, and a pericardial effusion (as shown on the echocardiogram). Major takeaway Myocardial infarction may be complicated by myocardial wall rupture 3-14 days after the initial infarction. This causes acute cardiac tamponade, which is characterized by Beck's triad: hypotension, jugular venous distension, and distant heart sounds. Main explanation This patient has developed cardiac tamponade (acute build-up of fluid in the pericardial space) five days after a myocardial infarction. This patient has all three features of the Beck triad of acute cardiac tamponade: hypotension, jugular venous distension, and distant heart sounds. Myocardial wall rupture occurs in about 3% of people following myocardial infarction, and most commonly occurs 3-14 days after the infarction. Pulsus paradoxus is an abnormally large decrease in systolic blood pressure and pulse wave amplitude during inspiration, and is a sign of acute cardiac tamponade. In this situation, urgent pericardiocentesis is required to drain the fluid around the heart and restore function. Histology shows extensive phagocytosis of dead cells, along with collagen deposition.

A 60-year-old man is reviewed on the cardiology ward because of hypotension and prominent jugular vein distention for 10 minutes. He is on the ward because of a large myocardial infarction five days ago. Medical history includes type 2 diabetes mellitus, uncontrolled hypertension, and hypercholesterolemia. His temperature is 36.8°C (98°F), pulse is 112/min, respirations are 21/min, and blood pressure is 94/78 mm Hg. His pulse is weaker to palpation during inspiration, and cardiac auscultation shows muffled heart sounds. Which of the following is the most likely diagnosis? A. Arrhythmia B. Myocardial infarction C. Myocardial wall rupture D. Pericarditis E. Ruptured papillary muscle

F. Deposition of calcium without luminal narrowing Major takeaway Mönckeberg sclerosis ( medial calcific sclerosis ) is an uncommon condition which leads to calcification of the internal elastic lamina and media (with sparing of the intima) of arteries. It is most often asymptomatic and can be identified as an incidental finding on x-ray. Main explanation This asymptomatic patient, without claudication and intact peripheral pulses, is found to have an incidental finding on x-ray of the hip, which most indicative of Mönckeberg sclerosis ( medial calcific sclerosis ). It is an uncommon condition caused by calcification of the internal elastic lamina and media (but spares the intima) of arteries, leading to vascular stiffening without luminal obstruction. It is often described as a "pipestem" appearance on x-ray. Typically, Mönckeberg arteriosclerosis is asymptomatic, unless complicated by atherosclerosis or calciphylaxis. The pathogenesis is not well understood, but the condition is thought to be the consequence of disordered calcium phosphate regulation in the blood vessel, leading to calcium phosphate crystals (hydroxyapatite) deposition. Mönckeberg sclerosis is a benign variant of arteriosclerosis, and the differences between different types of arteriosclerosis are shown in the table below:

A 60-year-old woman comes to the clinic due to hip pain that began after she slipped and fell on her side while going down the stairs 2 hours ago. Past medical history is significant for osteoarthritis. The patient has had recent difficulty walking due to hip pain but is otherwise healthy. Vitals are within normal limits. On physical exam, the right hip is tender to touch and peripheral pulses are intact. An x-ray of the lower extremities is obtained (shown below) and no fractures or displacements are seen. Which of the following is responsible for the appearance of this patient's femoral arteries? A. Intimal fatty streak formation B. Separation of the media and intima C. Proliferation of smooth muscle cells D. Inflammation of the media E. Deposition of hyaline F. Deposition of calF. Deposition of calcium without luminal narrowingcium without luminal narrowing

B. Papillary muscles Major takeaway Myocardial infarction may result in papillary muscle damage after the event. The damage occurs most commonly in the left ventricle and can cause mitral regurgitation. Main explanation Located in the ventricles of the heart, the papillary muscles contract before systole in order to hold the atrioventricular valves closed and prevent prolapse. There are five papillary muscles - one per valve cusp - that are connected to the cusps via chordae tendinae. Following myocardial infarction, there is potential for these muscles to become damaged or rupture due to ischemia. In this case, regurgitant valve disease can occur, most commonly mitral, as in this patient. Mitral regurgitation is characterized by the first heart sound being followed by a high-pitched holosystolic murmur at the apex, that radiates to the back or clavicular area. Mitral regurgitation may cause pulmonary edema due to increased pressure in lung capillaries. This is auscultated as basal lung crackles.

A 62-year-old Caucasian woman comes to the office because of shortness of breath for the past six hours. She had a myocardial infarction five days ago, which was treated medically because she failed to come to the hospital in time for primary percutaneous coronary intervention. On cardiac auscultation, a high-pitched holosystolic murmur is heard at the apex, when she is lying on her left side. Coarse crackles are heard at the lung bases bilaterally on pulmonary auscultation. Which of the following muscles is most likely damaged? A. Chordae tendinae B. Papillary muscles C. Pectinate muscles D. Pectoral muscles E. Right ventricular muscle

D. Abdominal aorta Major takeaway Atherosclerosis is characterized by intimal lesions called atheromas (atherosclerotic plaques) that occlude the vascular lumen of large- and medium-sized vessels. In descending order, the most common arteries affected by atherosclerosis are the abdominal aorta, coronary artery, popliteal artery, and then the carotid artery. Main explanation This patient presenting with substernal chest pain radiating to the left arm and ST-segment elevations in the inferior leads likely had a myocardial infarction. Subsequent autopsy showed yellow lipid plaques in the coronary arteries, suggestive of atherosclerosis. Atherosclerosis is a pathologic process that affects arteries throughout the body, including the aorta, coronary, cerebral, and peripheral arteries. Atherosclerosis is the hardening of any artery (but usually medium-large arteries) which is caused by the buildup of plaques (termed "atheromas"). Atherosclerosis can begin as early as childhood with the development of fatty streaks and advances with aging.In descending order, the most common arteries affected by atherosclerosis are: Abdominal aorta Coronary artery Popliteal artery Carotid artery Risk factors for atherosclerosis can be divided into modifiable and nonmodifiable risk factors. Modifiable risk factors include hypertension, diabetes mellitus, smoking and dyslipidemia, (particularly an increase in LDL levels or a decrease in HDL levels). Non-modifiable risk factors include age, family history, and being of African-American descent.

A 62-year-old man is brought to the emergency department by his partner after experiencing crushing substernal chest pain radiating to the left arm for the past hour. The patient reports experiencing nausea and diaphoresis during this time. An ECG is performed, which shows sinus rhythm with ST-segment elevation in leads II, III and aVF. Before cardiac catheterization could be performed, the patient develops ventricular tachycardia with subsequent hemodynamic instability. Despite appropriate life-saving measures, the patient dies. Autopsy shows yellow lipid plaques on the intimal surface of the right coronary artery with an overlying, completely occlusive thrombus. Examination of several other coronary arteries demonstrate similar lipid plaques. Which of the following locations is most likely to show similar lesions? A. Radial artery B. Internal carotid artery C. Femoral artery D. Abdominal aorta E. Subclavian artery

C Congestive heart failure Low specific gravity and normal glucose would be expected on this patient's pleural fluid analysis. Congestive heart failure is the most likely underlying cause of this patient's pleural effusion. In HF with reduced ejection fraction, there is an elevation in left-sided filling pressures due to left ventricular systolic dysfunction, which, in turn, results in elevated pulmonary capillary hydrostatic pressures and increased interstitial fluid in the lung. This fluid then permeates into the pleural cavity and manifests as unilateral or bilateral pleural effusion. CHF almost always results in a transudative pleural effusion, which is seen as clear, pale, yellow-colored fluid on thoracentesis.

A 63-year-old man comes to the physician because of a 3-week history of fatigue and shortness of breath. Physical examination shows diminished breath sounds at the right lung base. An x-ray of the chest shows blunting of the right costophrenic angle. Thoracentesis shows clear, yellow-colored fluid with a protein concentration of 1.9 g/dL. Which of the following is the most likely underlying cause of this patient's pleural effusion? A Pulmonary tuberculosis B Pulmonary sarcoidosis C Congestive heart failure D Chronic pancreatitis E Thoracic duct injury F Bacterial pneumonia

C. Regurgitant valve disease Major takeaway Myocardial infarction may result in regurgitant valvular disease as a result of papillary muscle damage. This may be auscultated as a systolic murmur. Main explanation This patient is having a myocardial infarction, which is characterized by central "crushing" chest pain which radiates to the jaw or left arm. Patients may also have tachycardia, hypotension, nausea, distress, and dizziness. A myocardial infarction can affect any of the papillary muscles in the ventricles. This means the muscles fail to get the blood supply they require to function effectively. As such, these muscles may no longer be able to maintain the position of the valve leaflets against rising ventricular pressure, enabling regurgitation (typically mitral). The characteristic murmur of mitral valve regurgitation is a holosystolic murmur that is loudest at the heart's apex and radiates to the axilla. The image shows a picture of the papillary muscles attached to the chordae tendinae ("heart strings"). This regurgitation may be auscultated as a systolic murmur.

A 64-year-old man comes to the emergency department because of severe central chest pain for 2 hours. He says the pain is heavy in nature and radiates to his jaw. He has a history of uncontrolled hypertension and type 2 diabetes mellitus. His temperature is 36.8°C (98°F), pulse is 97/min, respirations are 18/min, and blood pressure is 163/91 mm Hg. Cardiovascular auscultation shows a holosystolic murmur that seems loudest at the apex. ECG shows ST-elevation in leads V2, V3, and V4. Which of the following will most likely occur as a result of this pathology? A. Fibrotic valve disease B. Infectious valve disease C. Regurgitant valve disease D. Rheumatic valve disease E. Stenotic valve disease

C. Granulomatosis with polyangiitis Major takeaway Granulomatosis with polyangiitis (GPA) is an autoimmune condition that affects the lungs and kidneys. It is associated with cytoplasmic anti-neutrophil cytoplasmic antibodies (c-ANCA) and commonly causes nasal inflammation resulting in bloody or purulent nasal discharge. Main explanation Granulomatosis with polyangiitis (GPA), formerly referred to as Wegener's granulomatosis (WG), is a systemic disorder that involves both granulomatosis and polyangiitis and affects small- and medium-sized vessels in many organs. It is associated with cytoplasmic anti-neutrophil cytoplasmic antibodies (c-ANCA). Damage to the lungs and kidneys can be fatal and it requires long-term immunosuppression.In 1990, the American College of Rheumatology accepted the classification criteria for GPA. These criteria were not intended for diagnosis, but for inclusion in randomized controlled trials. Two or more positive criteria have a sensitivity of 88.2% and a specificity of 92.0% of describing GPA.Nasal or oral inflammation: painful or painless oral ulcers, or purulent or bloody nasal dischargeLungs: abnormal chest X-ray with nodules, infiltrates, or cavities.Kidneys: urinary sediment with microhematuria or red cell castsBiopsy: granulomatous inflammation within the arterial wall or in the perivascular areaThe condition was originally named for Friedrich Wegener, who described the disease in 1936. As a response to Wegener's association with the German Nazi party, professional bodies and journals have replaced his name with a descriptive name.

A 64-year-old woman comes to the emergency department because of sinusitis, a runny nose, and hemoptysis for the past 3 months. The nasal drainage is often purulent and occasionally contains blood. The patient has also experienced new joint pains and fatigue during the same time-period. Examination shows tenderness to percussion over the maxillary sinuses; however, is otherwise non-contributory. Urinalysis shows 3+ microscopic hematuria and 2+ proteinuria. Chest X-ray is shown below. What is the most likely diagnosis? A. Chronic heart failure B. Goodpasture syndrome C. Granulomatosis with polyangiitis D. Non-Hodgkin lymphoma E. Pneumonia

E. Unstable angina Major takeaway Acute coronary syndrome includes ST-elevation myocardial infarction (STEMI), non-ST-elevation myocardial infarction (NSTEMI) and unstable angina (UA). UA occurs when there is chest pain at rest, without ECG changes or elevations in cardiac biomarkers. Main explanation This patient has unstable angina , and this question requires that you know the definitions of ST-elevation myocardial infarction (STEMI), non-ST-elevation myocardial infarction (NSTEMI) and unstable angina (UA), which are the three types of acute coronary syndrome (ACS). Unstable angina is considered to be an ACS in which there is no detectable release of the enzymes and biomarkers of myocardial necrosisAn elevated troponin, would make the diagnosis NSTEMI and not UA. The main difference between UA and NSTEMI is that in NSTEMI the ischemia is severe enough to cause myocardial damage and a troponin leak (and thus in UA, elevated troponin is not seen). During NSTEMI, ST-depression can sometimes also be seen. Finally, STEMI is characterized by ST-elevation on ECG. There is also elevated troponin in STEMI. In non-ST- elevation myocardial infarction ( NSTEMI ) there would be an elevation in cardiac biomarkers such as troponin. This is because in NSTEMI the ischemia is severe enough to cause myocardial damage and a troponin leak In non-ST- elevation myocardial infarction ( NSTEMI ) there would be an elevation in cardiac biomarkers such as troponin. This is because in NSTEMI the ischemia is severe enough to cause myocardial damage and a troponin leak

A 65-year-old man comes to the emergency department because of severe central chest pain for two hours. He says the pain began while he was sitting at the breakfast table, is heavy in nature, and radiates to his jaw. He has a history of uncontrolled hypertension and diabetes mellitus type 2. He also has a history of cigarette smoking. His temperature is 36.8°C (98°F), pulse is 97/min, respirations are 18/min, and blood pressure is 163/91 mm Hg. Cardiovascular examination shows no abnormalities. ECG shows sinus rhythm at 97/min without ST-segment or other changes. Cardiac biomarkers taken six hours after admission are not elevated. Which of the following is the most likely cause of his symptoms? A. Duodenal ulcer B. Non-ST-elevation myocardial infarction C. ST-elevation myocardial infarction D. Stable angina E. Unstable angina

E. Smoking cessation, exercise, blood sugar, and hypertension control Major takeaway Intermittent claudication due to atherosclerosis and peripheral vascular disease should be first managed with conservative therapy. This will aid correction of the underlying pathologies. Main explanation Pain in the lower extremities that is aggravated by exercise and relieved with rest is highly indicative of claudication due to atherosclerotic disease. Atrophic changes such as hair loss, thickening of nails, and shiny skin are also supportive of this diagnosis.Vascular risk factors are an important clue to diagnosis. This patient is a current smoker, is hypertensive, and has diabetes mellitus. All of these factors damage or facilitate damage of vascular endothelium. This patient also has a prior history of myocardial infarction, which is another atherosclerotic disease. Claudication is "crampy" limb pain resulting from muscular ischemia during exercise, due to narrowed arteries.The best initial treatment for this condition is conservative therapy with smoking cessation, exercise, and control of diabetes mellitus, hypertension, and dyslipidemia.

A 65-year-old man comes to the office because of pain in the lower extremities. The pain is associated with walking and relieved by rest. He has a history of myocardial infarction, hypertension, hyperlipidemia, diabetes mellitus, and smoking. Examination of the legs shows atrophic changes with diminished pedal pulses. Which of the following is the most appropriate initial treatment? A. Clopidogrel B. Metoprolol C. Pedal pumping D. Revascularization of the extremities E. Smoking cessation, exercise, blood sugar, and hypertension control

C. Administration of prednisone Major takeaway Polymyalgia rheumatica affects patients greater than 50 years of age and causes proximal bilateral aching and morning stiffness, along with elevated erythrocyte sedimentation rate. This inflammatory condition can be treated with low-dose glucocorticoids. Main explanation This patient's symptoms of aching and morning stiffness in the neck and shoulders are consistent with polymyalgia rheumatica (PMR), a chronic inflammatory disease of unknown cause that typically affects individuals older than 50 years of age. PMR is characterized by pain and morning stiffness in the neck, shoulders, torso, and hips. Other symptoms include distal musculoskeletal manifestations like synovitis, tenosynovitis, and bursitis. Nonspecific features include fever, malaise, weight loss, and normocytic anemia. Laboratory studies show an elevated erythrocyte sedimentation rate. PMR can be treated with low-dose glucocorticoids. High-dose treatment is appropriate for patients with concurrent giant cell arteritis, to prevent the potential complication of vision loss. PMR is diagnosed clinically, but temporal artery biopsy should be performed in patients with suspected giant cell arteritis.

A 65-year-old woman comes to the office because of a 3-month history of neck and shoulder pain. She initially noticed symptoms only on the left side but has experienced bilateral pain and stiffness for the past two months. The stiffness is worst in the morning and lasts more than an hour. She reports a 4.5-kg (10-lb) weight loss during the past three months. Physical examination shows swelling of shoulders with limited range of motion. Muscle strength is normal in all extremities. Laboratory studies show an erythrocyte sedimentation rate of 70 mm/h. Which of the following is the most appropriate next step in management? A. Temporal artery biopsy B. X-rays of the shoulders C. Administration of prednisone D. Muscle biopsy E. Measurement of creatine kinase levels

C. Pneumonia Major takeaway Common symptoms of pneumonia include: cough, fevers, chills, fatigues, rigors and pleuritic pain. The diagnosis can often be made on history and examination alone. Key historical factors to explore include: recent respiratory illness, travel, smoking and occupational risks. Main explanation Cough, purulent sputum, fever , and fatigue are characteristic symptoms of pneumonia. Pneumonia may be community acquired, healthcare associated, or hospital acquired. It is important to establish whether there has been recent contact with health services to see which category patients fall into as the empirical treatment can differ.The diagnosis of pneumonia is generally made clinically through history and examination, with chest XR and bloods used to confirm the diagnosis. Key historical factors to explore include recent respiratory illness, travel, smoking, and occupational risks. Characteristic examination findings over the involved lung segment are an increase in tactile fremitus, dullness to percussion, course crepitations, increased vocal resonance, and egophony (an E-to-A change over consolidated lung).

A 66-year-old man comes to the emergency department because of a 2-day history of a productive cough with yellow sputum, chest tightness, fatigue, chills, and fever. He has smoked cigarettes since the age of 25. Vital signs show his temperature is 38.8°C (102°F), pulse is 80/min, respirations are 22/min, and blood pressure is 140/90 mm Hg. Auscultation of his lungs shows rhonchi over the lower 2/3 of the left lung and bilateral crackles at the bases. The patient is told to pronounce the long-E vowel sound while you auscultate the lungs, the E sounds like an A over the L lung. Tactile fremitus is increased in this area and the percussion note is dull. Which of the following is the most likely diagnosis? A. Bronchitis B. Atelectasis C. Pneumonia D. Lung cancer E. Tuberculosis

E. Unstable angina n non-ST- elevation myocardial infarction ( NSTEMI ), ischemia is severe and results in a detectable release of biomarkers of myocardial injury hours after the onset of ischemic chest pain, most commonly cardiac troponin I, troponin T, and the MB isoenzyme of creatine kinase. In unstable angina , there is no detectable enzyme increase.

A 66-year-old man comes to the emergency department because of left-sided chest pain that began at rest and lasted for 15 minutes before resolving. He says a similar episode occurred at rest yesterday. Medical history includes hypertension and type II diabetes mellitus. Current medications are amlodipine, glyburide, and aspirin. His temperature is 36.8°C (98°F), pulse is 87/min, respirations are 18/min, and blood pressure is 117/78 mm Hg. Cardiac examination shows carotid upstroke is normal, there are no cardiac murmurs, and the lung fields are clear. ECG shows ST-depression in leads V2, V3, and V4. Laboratory investigations show elevated CK-MB and serum troponin I level. Chest X-ray is normal. Which of the following is the most likely diagnosis? A. Chronic stable angina B. Non-ST-elevation myocardial infarction C. Pulmonary embolism D. ST-elevation myocardial infarction E. Unstable angina

E. C and D

A 67-year-old man comes to the emergency department because of fever and shortness of breath for the past 6 days. He has associated chills and a cough productive of multiple tablespoons of thick green sputum each morning. He has a history of diabetes and end-stage renal disease requiring regular dialysis which he gets at a center near his home. He lives at home with his wife, has had no recent travel or sick contacts, and no recent hospitalizations. His temperature is 38.5°C (101.3°F), pulse is 100/min, respirations are 22/min, and blood pressure is 100/72 mm Hg. Pulse oximetry on room air shows an oxygen saturation of 92%. He can speak in 3-4 word sentences. Physical examination shows bibasilar crackles on auscultation, and diminished breath sounds at the right middle and right lower lung fields. What are the next best steps? A. ECG B. ECHO C. Sputum Gram Culture D. Blood Cultures E. C and D F. None of the Above

D. Echocardiography Major takeaway Pericardial effusions can result from underlying malignancy and can cause sinus tachycardia, low QRS voltage, and electrical alternans on ECG. Echocardiography is the test of choice for confirming the diagnosis and assessing the extent of a hemodynamic compromise. Main explanation Multiple aspects of this patient's history, physical examination, and diagnostic studies are suggestive of an acute pericardial effusion . Pericardial effusions can result from a variety of causes, one of which is a malignancy. Other causes include viral or infectious pericarditis, autoimmune disease, and recent myocardial infarction. The classic signs on physical examination are hypotension, muffled heart sounds, and jugular venous distention. On ECG, pericardial effusion typically manifests with sinus tachycardia, low QRS voltage, and electrical alternans. Electrical alternans (shown with arrows) refers to a cyclic shift in the QRS axis, due to the mechanical swinging of the heart. In a patient with suspected pericardial effusion, echocardiography is considered the definitive test for establishing the diagnosis and is both sensitive and specific for pericardial effusion. In addition, echocardiography can provide information regarding the hemodynamic effects of the effusion, including the need for interventions such as

A 67-year-old man with a recent diagnosis of lung cancer is admitted to the hospital because of worsening cough, chest pain, and dizziness. His pulse is 140/min, respirations are 26/min, and blood pressure is 80/50 mm Hg. He has smoked a pack of cigarettes daily for 40 years and has a 20-year history of chronic obstructive pulmonary disease. Cardiac examination shows muffled heart sounds and no jugular venous distention. An ECG is obtained and is shown below. Which of the following is the most appropriate next step in diagnosis? A. CT scan of the chest B. Chest X-ray C. Complete blood count D. Echocardiography E. MRI of the chest

E. Nitroglycerin Major takeaway Nitroglycerin (and other nitrates) are contraindicated in patients with myocardial infarction if a phosphodiesterase inhibitor has been taken in the last 24 hours, due to the risk of inducing severe hypotension. Main explanation The patient is likely experiencing acute coronary syndrome and needs immediate attention. Nitroglycerin is contraindicated in this patient because of the recent intake of sildenafil, a phosphodiesterase type 5 inhibitor, in the last 24 hours. The administration of nitroglycerin can precipitate severe hypotension. The main mechanism of action of nitrates involve decreasing cardiac preload by venodilation, and a minor decrease in afterload by vasodilation of arterial beds. This is achieved by promoting the conversion of guanosine triphosphate (GTP) to cyclic guanosine monophosphate (cGMP), which causes smooth muscle relaxation via Protein Kinase G. Sildenafil inhibits phosphodiesterase type 5, the enzyme necessary for the breakdown of cGMP, thereby preventing the breakdown of nitrate-mediated increases in cGMP, causing excessive vasodilation and hypotension.

A 68-year-old man comes to the emergency department because of 4 hours of acute onset retrosternal chest pain, radiating to the left arm. It began while he was playing soccer with his grandson. His past history is significant for type 2 diabetes mellitus controlled with insulin, and hypertension controlled with enalapril. Further history revealed that he had ingested a sildenafil tablet yesterday. Physical examination shows a temperature of 37°C (98.6°F), pulse of 69/min, respirations of 18/min, blood pressure of 130/70 mm Hg, and O2 saturation of 95%. An electrocardiogram shows sinus rhythm and ST-segment elevation in leads V4-V6. Which of the following medications is contraindicated for use in this patient for treating his symptoms? A. Aspirin B. Atenolol C. Enoxaparin D. Morphine E. Nitroglycerin

D. Non-small-cell lung carcinoma Major takeaway Primary lung cancer is characterized by the Golden S sign, which is commonly seen in right upper lobe collapse. The Golden S sign occurs when the pleural edge takes on a reverse S shape. Main explanation The chest X-ray shows increased density in the upper medial hemithorax with a loss of volume and a shift of the trachea to the right. A mass is present at the right hilum. The right hilar mass (orange) obstructing the right upper lobe bronchus results in the collapse of the right upper lobe (green arrow). This results in a reverse S shape to the pleural edge: the Golden S sign.The Golden S sign also resembles a reverse S shape on posteroanterior (PA) chest radiographs and therefore may also be referred to as the reverse S sign of Golden. The Golden S sign can be seen with the collapse of all lobes of the lung but is most commonly seen in right upper lobe collapse. It is created by a central mass obstructing the upper lobe bronchus and should raise suspicion of a primary bronchogenic carcinoma, of which about 80% are non-small-cell lung carcinomas. These include squamous cell carcinomas, which account for 30% of lung cancers and typically occur centrally.

A 69-year-old man comes to the office because of shortness of breath for the past 3 months. He has also lost 3-kg (6.6-lb) and had occasional blood-stained sputum. He has a history of hypertension, type 2 diabetes mellitus, and gout for which he takes enalapril, metformin, and allopurinol. Pulmonary examination shows no abnormalities. A chest X-ray is obtained and is shown below. Which of the following is the most likely diagnosis? A. Atelectasis B. Bacterial pneumonia C. Pleural effusion D. Non-small-cell lung carcinoma E. Small-cell lung carcinoma

A. Aortitis Major takeaway Aortitis means inflammation of the aorta and has different etiologies, divided into infectious and noninfectious etiologies, which have respectively characteristic symptoms. The patient here has giant cell arteritis, or inflammation of the temporal arteries which is a common noninfectious etiology of aortitis. Her symptoms are consistent with this disease. Main explanation This patient's symptoms are due to aortitis, or inflammation of the aorta. This patient suffers from giant cell arteritis, or inflammation of the temporal arteries. This is a common cause of aortitis which is a clue to the diagnosis. Aortitis can be classified into infectious and noninfectious etiologies, and the patient in this case can attribute her aortitis to giant cell arteritis (a noninfectious etiology). Thus, additional vessels, such as temporal arteries, are affected as well as the aorta, accounting for her throbbing headache. The fatigue, fever, and claudication she is experiencing are common symptoms of aortitis due to giant cell arteritis. Bruits are a common finding in aortitis as vessels are stenosed due to thickening from inflammation. The high ESR and CRP are consistent with giant cell arteritis. Finally, the high signal intensity on the MRI suggests inflammation of the aorta.

A 69-year-old woman comes to the emergency department because of a throbbing headache, fatigue, a fever, and polymyalgia rheumatica, specifically claudication of her jaw and arms, for the past day. Patient's medical history includes common headaches due to temporal arteritis. Physical examination shows that heart rate/rhythm and blood pressure are within normal ranges. Bruits are heard over the carotid arteries on auscultation. Laboratory results show that the erythrocyte sedimentation rate (ESR) and C-reactive protein (CRP) are elevated. Magnetic resonance imaging (MRI) scan shows a high signal intensity within the aortic wall. Which of the following is the most likely explanation for this patient's findings? A. Aortitis B. Acute pericarditis C. Angina pectoris D. Aortic coarctation E. Myocardial infarction

E. Phase 4 Main explanation This question essentially tests the relation of the electrical activity of the heart to its mechanical function. The coronary arteries fill up during diastole, as the ventricles relax. Therefore a stenotic lesion in a coronary artery will be best visualized on diastole, because during that period the dye will fill the coronary circulation . Relaxation of the ventricle corresponds to Phase 4, the resting membrane potential , during which cardiac myocytes are highly permeable to potassium. The cell remains in this phase until it is stimulated by an electrical stimulus, causing it to depolarize again. A is incorrect. Phase 0 is the upstroke of the action potential, corresponding to rapid depolarization of myocytes. Voltage-gated sodium channels open at that time. Shortly after depolarization, the ventricle starts to contract. B is incorrect. Phase 1 of the action potential occurs with the inactivation of the fast Na+ channels. Voltage-gated K+ channels begin to open. C is incorrect. Phase 2 is the plateau phase, sustained by a balance between inward movement of calcium through L-type calcium channels and outward movement of potassium. D is incorrect. Phase 3 is the "rapid repolarization" phase, corresponding to closure of Ca2+ channels, and massive efflux of K+ through voltage-gated K+ channels that are still open. This is associated with the T wave not resting.

A 70 year old man was brought to the hospital by ambulance subsequent to chest pain that lasted 40 minutes. In order to assess the degree of stenosis of his coronary arteries, he underwent coronary angiography, a procedure during which a contrast dye was injected in his coronary vessels. To which phase of the ventricular action potential does visualization of the stenotic lesion correspond? A. Phase 0 B. Phase 1 C. Phase 2 D. Phase 3 E. Phase 4

E. Troponin I Major takeaway Myocardial infarction is characterized by necrosis of heart tissue as a result of ischemia. Troponin I is sensitive and specific for myocardial infarction and levels rise within 2-3 hours post-MI, peak at about 2 days, and continues to stay elevated for about 7 days. Main explanation Troponin I is an enzyme that's useful in evaluating a myocardial infarction (MI). Troponin I levels begin to rise within 2-3 hours post-MI, peak at about 2 days, and continue to stay elevated for about 7 days. Troponin is a complex of three regulatory proteins (troponin C, troponin I, and troponin T) that is integral to muscle contraction in skeletal muscle and cardiac muscle, but not smooth muscle. Subtypes of troponin (I and T) are very sensitive and specific indicators of damage to the heart muscle (myocardium). They are measured in the blood to differentiate between unstable angina and myocardial infarction (heart attack) in people with chest pain or acute coronary syndrome. A person who recently had a myocardial infarction would have an area of damaged heart muscle and elevated cardiac troponin levels in the blood. This can also occur in people with coronary vasospasm, a type of myocardial infarction involving severe constriction of the cardiac blood vessels.

A 71-year-old woman comes to the emergency department because of severe central chest pain for 30 minutes this morning. She says the pain was cramping in nature and radiated down her left arm. She has a history of atrial fibrillation and type 2 diabetes mellitus. Her temperature is 36.8°C (98°F), pulse is 97/min, respirations are 18/min, and blood pressure is 163/91 mm Hg. Cardiovascular examination shows no abnormalities. ECG is obtained and is shown below. Which of the following biochemical measures would most likely be elevated and remain elevated for a week after this acute event? A. Alanine aminotransferase B. Aspartate transaminase C. Creatinine kinase-MB D. Lactate dehydrogenase (LDH) E. Troponin I

A. Acute inferior myocardial infarction Major takeaway Myocardial infarction may cause abdominal pain rather than chest pain, especially in inferior acute myocardial infarction. An ECG is helpful to determine this diagnosis. Main explanation There are ST-elevations and Q-waves apparent in leads II, III, and aVF -- indicative of an acute inferior wall myocardial infarction (MI). While chest pain or pressure is classic, inferior MIs are notorious for sometimes being characterized by abdominal pain, which results from irritation of the diaphragm. They may also cause nausea, vomiting, dizziness, and weakness. ST-elevations occur because of transmural injury. This results in systolic and diastolic currents of injury into the damaged tissue, which is seen on ECG as ST-elevation. Treatment for inferior myocardial infarctions is similar to that of other MIs, with the caveat that it is important to preserve preload. Inferior MIs may involve right ventricular damage and/or dysfunction, and a decrease in preload can lead to severe hypotension. Thus, opiates and nitrates should be avoided if one thinks that a patient is having an inferior MI.

A 72-year-old man comes to the emergency department because of severe upper abdominal pain for the past two hours. He says the pain is heavy in nature, and that he has vomited twice. Medical history includes uncontrolled hypertension and type II diabetes mellitus. His temperature is 36.8°C (98°F), pulse is 97/min, respirations are 18/min, and blood pressure is 163/91 mm Hg. Cardiovascular examination shows no abnormalities. ECG is obtained and is shown below. Which of the following is the most likely diagnosis? A. Acute inferior myocardial infarction B. Perforated duodenal ulcer C. Pericarditis D. Posterior myocardial infarction E. Pulmonary embolism

D. Myocardial rupture Major takeaway The most common cause of myocardial rupture is a myocardial infarction. Myocardial rupture is characterized by a laceration or tearing of the wall of the ventricles or atria of the heart. Patients classically present with acute hemodynamic deterioration and sudden death. Main explanation Myocardial rupture is characterized by a laceration or tearing of the wall of the ventricles or atria of the heart. The most common cause of myocardial rupture is a recent myocardial infarction (MI), with the rupture typically occurring 3 to 5 days after infarction. Other causes, may include cardiac trauma, endocarditis, cardiac tumors, and aortic dissection.Myocardial rupture occurs in 2-3% of patients with transmural myocardial infractions. This condition can be broadly divided into 2 categories: (1) acute or sudden (2) subacute or progressive. Acute myocardial rupture is characterized by a severe cardiac tamponade and sudden death. Subacute myocardial rupture is characterized by the progressive formation of a wall hematoma, thus leading to pericardial effusion. Risk factors involved in myocardial rupture, include female sex, advanced age, first MI, and hypertension.Patients will present with acute hemodynamic deterioration (i.e., hypotension, abnormal heart rate, cold extremities, peripheral cyanosis). On gross pathology, a myocardial infarction (3-7 days after) will show a yellowish lesion with hyperemic borders. Hemopericardium is a common finding in patients who suffered from a left ventricular free wall rupture. Most patients with a myocardial rupture involving the left ventricle free wall, die immediately. Mainstay therapy is an urgent surgical correction.

A 72-year-old woman is rushed to intensive care unit because of sudden hemodynamic instability. The patient is currently on the 3rd day of her hospital stay. Her medical history is relevant for a recent primary percutaneous coronary intervention due to a myocardial infarction. She currently takes dual antiplatelet therapy and statins. Physical exam shows pale and cold extremities and peripheral cyanosis. Her temperature is 35.1°C (95.1°F), pulse is absent, respirations are irregular, and blood pressure is 50/30 mmHg and pulse oximetry on room air shows an oxygen saturation of 75%. After a cardiac arrest and several cycles of cardiopulmonary resuscitation, the patient dies 30 minutes later. The autopsy revealed hemopericardium along with necrotic myocardium (a yellowish lesion with hyperemic borders). Which of the following is the most likely cause of death in this patient? A. Coronary artery dissection B. Rupture of chordae tendineae C. Takotsubo cardiomyopathy D. Myocardial rupture E. Ventricular aneurysm

A. Coronary artery disease Major takeaway Coronary artery disease, is a group of diseases that includes: stable angina, unstable angina, myocardial infarction, and sudden cardiac death. The first sign is occasionally a heart attack, and important clues to diagnosis are coronary risk factors as well as ECG findings. Main explanation This patient is having a non-ST-segment elevation myocardial infarction (NSTEMI). This is evidenced by the ECG showing widespread T-wave inversion due to myocardial ischaemia (which is most prominent in the lateral leads). NSTEMIs do not cause ST-elevation because the ischemia does not affect the entire cardiac wall.Coronary artery disease, is a group of diseases that includes: stable angina, unstable angina, myocardial infarction, and sudden cardiac death. Usually, symptoms occur with exercise or emotional stress, last less than a few minutes, and get better with rest. Shortness of breath may also occur and sometimes no symptoms are present. The first sign is occasionally a heart attack (as in this case).Risk factors for coronary artery disease include high blood pressure, smoking, diabetes, lack of exercise, obesity, high blood cholesterol, poor diet, and excessive alcohol consumption. The underlying mechanism involves atherosclerosis of the arteries of the heart. Remember to have a broad differential diagnosis with chest pain however, as there are many pathologies which can mimic the symptoms of coronary artery disease such as the other options listed in the question.

A 73-year-old male comes to the office because of chest pain for 2 hours. He returned to America yesterday from a holiday in New Zealand. He states that the pain feels heavy and is located behind his sternum. He also feels nauseated, and as a result hasn't eaten lunch. His medical history includes type II diabetes mellitus, dyslipidemia, and hypertension. His temperature is 36.5°C (97.7°F), pulse is 78/min, respirations are 17/min, and blood pressure is 120/78 mm Hg. Examination shows a man who appears pale and diaphoretic. Cardiovascular and respiratory examinations are noncontributory. An ECG is obtained. Which of the following is the most likely diagnosis? A. Coronary artery disease B. Pericarditis C. Gastric ulcer D. Pneumonia E. Pulmonary embolism

D. Parietal pleura Major takeaway Pneumothorax is characterized by a linear shadow of visceral pleura with a lack of lung markings peripheral to the shadow, indicating a collapsed lung on chest X-ray. In order for this to form the integrity of the pleural cavity needs to be breached. Main explanation The patient has a massive left sided tension pneumothorax . In this particular case, the pleural space was penetrated by the knife which allowed air to enter but not to exit. The resulting build up of air has caused pressure to build up in the pleural space resulting in a collapsed lung and mediastinal shift.This is an emergency condition. The buildup of pressure in the chest cavity can collapse the vena cava causing a complete circulatory collapse in minutes. Needle decompression followed by a chest tube is indicated.It is probable that the knife entered deeper layers including the visceral pleura or even the lung parenchyma; however, the point of this question is to realize that for a pneumothorax to form the integrity of the pleural cavity needs to be breached.

A 73-year-old woman comes to the emergency department because she sustained multiple stab wounds from a knife to the chest during a mugging 20 minutes ago. She says the pain is worse on inspiration and that she feels shortness in her breath. Her temperature is 36.8°C (98°F), pulse is 104/min, respirations are 26/min, and blood pressure is 95/78 mm Hg. Chest X-ray is obtained and is shown below. Which of the following is the deepest layer of tissue that the offending weapon penetrated? A. Dermis B. Epidermis C. Intercostal muscle D. Parietal pleura E. Pectoral muscle

A Erythrocyte sedimentation rate of 110 mm/hr

A 74-year-old man has had increasingly severe headaches for 2 months, centered on the right. His vital signs include T 36.9°C, P 82/minute, RR 14/minute, and BP 130/85 mm Hg. There is a palpable tender cord-like area over his right temple. His heart rate is regular with no murmurs, gallops, or rubs. Pulses are equal and full in all extremities. A biopsy of this lesion is obtained, and microscopic examination reveals a muscular artery with luminal narrowing and medial inflammation with lymphocytes, macrophages, and occasional giant cells. He improves with a course of high-dose corticosteroid therapy. Which of the following laboratory test findings is most likely to be present with this man's disease? A Erythrocyte sedimentation rate of 110 mm/hr B Rheumatoid factor titer of 80 IU/mL C HDL cholesterol of 15 mg/dL D Anti-double stranded DNA titer of 1:1024 E pANCA titer of 1:160

B. Inferior ST-elevation myocardial infarction Major takeaway Myocardial infarction , specifically inferior ST-elevation myocardial infarction should be suspected when an ECG shows ST-elevation in leads II, III and aVF, Q-wave formation in leads III and aVF, and reciprocal ST-depression and T-wave inversion in aVL. Main explanation This patient is having an inferior ST- elevation myocardial infarction (STEMI). Diabetic patients and women can often be characterized by more subtle symptoms and not classical crushing chest pain. They often complain of fatigue, nausea, and jaw pain/numbness.Inferior myocardial infarctions (MI) account for 40-50% of all myocardial infarctions. They have a more favourable prognosis than anterior myocardial infarctions with in-hospital mortality being only 2-9%, however certain factors indicate a worse outcome. Up to 40% of patients with an inferior STEMI will have a concomitant right ventricular infarction. These patients may develop severe hypotension in response to nitrates and generally have a worse prognosis. Up to 20% of patients with inferior STEMI will also develop significant bradycardia due to second- or third-degree AV block. Bradycardia may be due to occlusion of the branch of the right coronary artery which supplies the sinoatrial node. These patients have an increased in-hospital mortality (>20%). Inferior STEMI may also be associated with posterior infarction, which confers a worse prognosis due to increased area of myocardium at risk.

A 74-year-old woman comes to the emergency department because of severe "heartburn" for 40 minutes. In addition to her "heartburn," she says that she feels very fatigued and feels numbness in her jaw. She has vomited twice in the last 30 minutes. She has a history of insulin dependent diabetes mellitus and hypercholesterolemia. An ECG is obtained and is shown below. Which of the following is the most likely diagnosis? A. Anterior ST-elevation myocardial infarction B. Inferior ST-elevation myocardial infarction C. Lateral ST-elevation myocardial infarction D. Non ST-elevation myocardial infarction E. Posterior ST-elevation myocardial infarction

E. Labetalol Major takeaway Hyperplastic arteriolosclerosis has a characteristic "onion skin" appearance on biopsy of arterioles. It is associated with hypertensive emergencies, which are treated with multiple antihypertensive medications. Main explanation This patient has hyperplastic arteriolosclerosis, confirmed by biopsy. Arteriolosclerosis simply means hardening of the arterioles, of which atherosclerosis is the most common subtype. The biopsy shows the characteristic "onion skin" lesion of hyperplastic arteriolosclerosis, indicative of a severe, relatively acute increase in blood pressure. This increase in blood pressure is known as a hypertensive emergency (previously "malignant hypertension"). It is defined as hypertension (systolic >180 mm Hg or diastolic >120 mm Hg) with acute impairment of one or more organ systems (especially the central nervous system, cardiovascular system and/or the renal system). Treatment for a hypertensive emergency consists of multiple antihypertensive agents. The choice of agent depends on the etiology of the hypertensive crisis, the severity of the elevation in blood pressure, and the usual blood pressure of the patient before the hypertensive crisis. In most cases, the administration of an intravenous sodium nitroprusside injection is best because it has an almost immediate antihypertensive effect. Often, it is unavailable, so in less urgent cases, oral agents like captopril, clonidine, labetalol, or prazosin can be used, but all have a delayed onset of action (by several minutes)

A 75-year-old man comes to the primary care clinic because of tea colored urine for the past 2 weeks. He has been feeling more fatigued for the last few weeks, but denies any other recent changes. He has a history of hyperlipidemia and insulin-dependent diabetes mellitus, as well as a 40-pack-year smoking history. His blood pressure in the room is 201/95 mm Hg. Urinalysis shows 3+ protein and red blood cells. A renal biopsy is obtained and is shown below. Which of the following is the most appropriate medication for this patient? A. Albumin B. Cisplatin C. Dexamethasone D. Epoetin alfa E. Labetalol

B. Hyperplastic arteriosclerosis Major takeaway Hyperplastic arteriolosclerosis has a characteristic "onion skin" appearance on biopsy of arterioles. It is associated with hypertensive emergencies. Main explanation This patient with a history of uncontrolled hypertension now presents with extremely high blood pressure (>180/120) and signs of end-organ damage (e.g., confusion, headache, blurry vision). Along with the laboratory results (i.e., elevated creatinine and hematuria), this presentation is consistent with malignant hypertension, also known as hypertensive emergency. Hypertensive emergencies are often the result of hyperplastic arteriolosclerosis, which is characterized by concentric thickening ("onion skinning") of the arteries and arterioles due to smooth muscle proliferation. In hypertensive emergency, these changes are acutely accompanied by fibrinoid deposits and vessel wall necrosis (necrotizing arteriolitis), which are more pronounced in the kidney. The result is a marked narrowing of renal arterioles and small arteries, which causes significantly reduced blood flow and subsequent ischemia, resulting in acute kidney failure. Arteriosclerosis literally means "stiffening of the arteries," and the differences between different types of arteriosclerosis are shown in the table below:

A 75-year-old man is brought to the emergency department due to sudden-onset confusion and headaches accompanied by nausea and vomiting. The patient has a history of poorly controlled hypertension and hyperlipidemia due to noncompliance with medications. The patient has smoked a pack of cigarettes daily for 40 years. The patient's temperature is 37.0°C (98.6°F), pulse is 100/min, and blood pressure is 205/140 mmHg. On physical examination, the patient is dyspneic. Fundoscopic examination reveals bilateral papilledema. Laboratory results are obtained and shown below: Which of the following histologic findings would likely be seen if this patient's renal artery was biopsied? A. Monckeberg Arteriosclerosis B. Hyperplastic arteriosclerosis C. Hyalinae arteriosclerosis D. Atherosclerosis

C. Thoracentesis Major takeaway Pleural effusions are locations of fluid build-up within the lungs. Thoracentesis can diagnose the underlying causes of the pleural effusion and should be performed. Main explanation Unilateral pleural effusions have several causes including malignancy, cylothorax, hemorrhage, and infection. Radiologically, it can be identified by radiograph. Typical findings include the presence of a horizontal fluid line within one or more fissures as well as opacity in the region of the effusion. A thoracentesis should be performed to determine the underlying etiology. During this procedure a needle is advanced into the posterior chest and a sample of the effusion is taken. Cytology and labs will be able to determine the cause of the effusion. Thoracentesis can also be therapeutic, as draining the fluid from the chest cavity can decrease a patients respiratory distress.

A 75-year-old woman presents to the emergency room for progressive shortness of breath. History is positive for a recent bout of pneumonia for which she had been prescribed appropriate antibiotics. The patient admits that she has been mostly non-compliant with taking the medication. A chest X-ray is performed and is displayed below. Which of the following interventions would most likely give you the etiology of the patient's pathology? A. CBC B. CT of the chest C. Thoracentesis D. PET E. Pericardiocentesis

(A) Cardiac tamponade Answer A (correct answer): This patient has signs and symptoms of aortic dissection. This condition is caused by a tear in the intimal layer of the aorta which allows blood to enter the space between the intima and the media. When this occurs, blood can "dissect" (i.e. separate) the two layers and disrupt blood flow through the aorta and into large arteries. Classic clinical findings are chest pain radiating to the back, widened mediastinum on imaging and blood pressure differential between the left and right arms. Aortic dissection may lead to a number of complications. Disruption of carotid artery blood flow may cause stroke. Disruption of coronary artery blood flow may cause ischemia, most commonly involving the right coronary artery. Widening of the aorta may disrupt aortic valve closure leading to aortic regurgitation and acute heart failure. Blood may enter the pericardial space causing cardiac tamponade. Answer B (incorrect answer): Aortic regurgitation but not aortic stenosis is caused by aortic dissection. Answer C (incorrect answer): Ventricular septal rupture may occur following ST-segment elevation myocardial infarction, usually of the left anterior descending artery. Answer D (incorrect answer): Left atrial thrombus formation occurs in patients with atrial fibrillation and other atrial arrhythmias. Answer E (incorrect answer): Pulmonary embolism occurs in a number of hypercoagulable states.

A 75-year-old woman presents with chest pain. She has a history of hypertension for which she takes amlodipine and lisinopril. One hour ago she noted the abrupt onset of chest pain. Blood pressure is 154/88 mmHg in the left arm and 122/70 mmHg in the right. Pulse is 110/min. Chest X-ray shows a widened mediastinum. This woman is at the highest risk for which of the following? (A) Cardiac tamponade (B) Aortic stenosis (C) Interventricular septum rupture (D) Left atrial thrombus formation (E) Pulmonary embolism Submit


Related study sets

Cultural Anthropology Final 2020

View Set

United States Lesbian and Gay History Final

View Set

Chapter 7: Instrumental Conditioning - Motivational Mechanisms

View Set

Business Practices Quiz # 2 missed questions

View Set

Integumentary Medications NCLEX Saunders

View Set

MICRO CH 15 - Wage Determination

View Set